You are on page 1of 367

.. , ..

.. , ..


,
010700.62
010701.65

. ..
2010

. . , . . , . . ,
. . . . /
. .: , 2010. 368 .
ISBN 978-5-8279-0084-9

.
.
, , .
,
, ,
, .


.
: ..-. ., .. ,
..-. ., ..

ISBN 978-5-8279-0084-9


. .. , 2010 .
, 2010 .



. ..
,
.
: , , .
, : ,
. , .

. , : , , .
, , . , .
, , ,
, . .
,
.

.
.
, . , ,
.
. .. .

................................................................................................... 7
1.
................................................................................................ 9
1.1. ................................................................. 9
1.2. ............................... 16
1.2.1. ...................................... 16
1.2.2. ........................... 16
1.3. ............................................................... 17
1.4. ......................................... 39
2.
.............................................................................................. 45
2.1. ............................................................... 45
2.1.1. ................................................................... 45
2.1.2. , ..... 46
2.2. ............................... 51
2.2.1. .......................................... 51
2.2.2.
.................................................................. 51
2.3. ................................................................ 52
2.4. ........................................ 75
3.
..................................................... 80
3.1. . .............................................................. 80
3.1.1. ......................................... 80
3.1.2. .............................................................................. 83
3.1.3. ........................................... 87
3.1.4. ................................................................. 89
3.2. ............................... 89
3.2.1. ........................................................... 89
3.2.2. ................................................ 90
3.3. ............................................................... 91
3.4. ...................................... 112
4.
. ............................................ 116
4.1. ............................................................. 116
4.2. ............................. 119
4.2.1. ......................................................... 119
4.2.2.
.... 120

4.3. ............................................................. 121


4.4. ...................................... 142
5. ................................. 146
5.1. ............................................................. 146
5.1.1. ()
.................................................................. 146
5.1.2. ................................................... 147
5.1.3. ................................. 149
5.1.4. - ............................ 151
5.1.5. () ............................ 154
5.2. ............................. 155
5.2.1. ..................................................................................... 155
5.2.2.
.................................................................. 156
5.3. ............................................................. 157
5.4. ...................................... 180
6. ............. 184
6.1. ............................................................. 184
6.1.1. .............................. 184
6.1.2. ................................. 190
6.2. .............................. 198
6.2.1. .................................................................. 198
6.2.2.
.................................................... 199
6.3. ............................................................. 200
6.3.1. .............................. 200
6.3.2. ................................. 208
6.4. ...................................... 226
7. .
. . ...................................... 231
7.1. ............................................................. 231
7.2. ............................. 237
7.2.1. ......................................................... 237
7.2.2. .............................................. 237
7.3. ............................................................. 238
7.3.1. .. 238
7.3.2. . .................................. 258
7.4. ...................................... 266

8.
. ..................................................... 270
8.1. . ............................................................ 270
8.1.1. .......................... 271
8.1.2. ............................... 277
8.1.3. . ................................. 281
8.2. ............................. 284
8.2.1. ......................................................... 284
8.2.2. .............................................. 285
8.3. ............................................................. 285
8.4. ...................................... 322
9. . . 326
9.1. ............................................................. 326
9.1.1. ........................................................... 326
9.1.2. .................................................................... 327
9.1.3. ............................................................ 329
9.1.4.
................................................................................... 331
9.1.5. ........................................... 336
9.1.6. () ................. 338
9.1.7. ........... 339
9.2. ............................. 344
9.2.1. ......................................................... 344
9.2.2. .............................................. 345
9.3. ............................................................. 346
9.4. ...................................... 362
................................................................................................. 365

. . . .
, , ,

.
,
. ,
.
, , . :
1)
;
2)
, , .
,
.
.
,
:
1) ,
2) ,
3) ,
4) .
,
, , ,

.
. ,
, .

, ,
.

. 10 12 10 12 . .
,
, . .

. .
, ,
. , ,
, .
, .

1.

1


1.1.

().

.
.
( ) (.), ,
, ().
. , ,
, ,
, ,
.
,

,
.
,
.

, .
1, , , .
A B, (
):
1
, ,
.

10

t1A + t 2A
.
(1.1)
2
A -

tB =
t1A

( ) A, t B B B, t 2A
A B
A.
(, ), , ( )
.
( ) S
- r .
- r ,
, (. . 1.1):
r = xi + yj + zk = {x, y , z} ,
(1.2)
i , j k : i = 1 , j = 1 ,
k = 1 ; x, y, z .
Z

Z
M

z(t)

r (t )
r (t + t )
x(t)
X

r (t )

r (t )

y(t) Y

(t )

. 1.1. - r (t ) , r (t + t ) () (t ) ()


- :

1.

11

r = r (t );
x = x(t ),

y = y (t ),
z = z (t ).

(1.3)

, -
.

F1 ( x, y, z ) = 0,
(1.4)

F2 ( x, y, z ) = 0,
,
(1.3). , , .
r (t ) - t t
(. 1):
r (t ) = r (t + t ) r (t ) =
= {x(t + t ) x(t ), y (t + t ) y (t ), z (t + t ) z (t )} .
(1.5)

, - (
,
):
d r (t )
r (t )
(t ) = { x (t ), y (t ), z (t )}
r&(t ) lim
=
t 0 t
dt
= {x& (t ), y& (t ), z&(t )} ,
(1.6)
x , y , z

.
:
(1.7)
(t ) = x& (t ) i + y& (t ) j + z&(t ) k = x (t ) i + y (t ) j + z (t ) k .

(t ) = x2 + y2 + z2 .

(1.8)

12


(. . 1.1).
(t ) ,
- r0 r (t0 ) t0,
:
t

r (t ) = r0 + r (t )dt .

(1.9)

t0

s(t),
( ) t,
t

s (t ) = (t )dt ,

(1.10)

(t )
d s(t )
( t ) ( t ) =
= s&(t ) .
(1.11)
dt
a
, ( ):
a(t ) = {a x (t ), a y (t ), a z (t )} &(t ) = {& x (t ),& y (t ),& z (t )},
(1.12)
ax, ay, az a . a
:
(1.13)
a(t ) = &x (t ) i + & y (t ) j + &z (t ) k = a x (t ) i + a y (t ) j + a z (t ) k .
a
a(t ) = a x2 + a 2y + a z2 .

(1.14)

a (t ) ,
0 (t0 ) - r0 r (t0 )
t0,
:
t

(t ) = 0 + a (t ) d t ,
t0

(1.15)

1.

13

t ''

r (t ) = r0 + 0 (t t0 ) + a (t ' ) d t ' d t ' ' .


(1.16)

t0 t0

- t0 :
r (t0 ) = r0 ,
(1.17)

(t0 ) = 0 .
a
a (. . 1.2):
(t ) d r
(t )
=
, (t ) = 1 , a (t ) = a (t ) (t ) ,
(1.18)
(t ) d s
d (t )
a (t ) =
= & (t ) ,
(1.19)
dt
a (t ) a .
t

r (t )

n(t )

a n (t )

a (t )
a (t )

(t )

. 1.2. a a
an

a (t ) > 0 ,
a (t ) < 0 , a (t ) = 0 ,
a (t ) = const 0 .
an a ,
(. 1.2):
an (t ) = an (t )n(t ) , n(t ) (t ) , n(t ) = 1 ,
(1.20)
an (t ) a n , .

(

14

), ,

2 (t )
0,
(1.21)
an (t ) =
(t )
ds
(t )
,
d
d t t + dt.
a
an a :
a (t ) = an (t ) +a (t ) .
(1.22)
a

a(t ) = an2 (t ) + a2 (t ) .
(1.23)
(1.21) (1.22)

, (. . 1.2).

Y
(t )
,
.. d
R

(t)
(t ) = R (. 1.3),
Z
X
(t )
(t ) :
(t )
d ( t )
. 1.3. (t )
& (t ) =
,

dt
R
(1.24)

a
(
t
)

(
t
)

(t ) & (t ) =
=
.
R
R
:
2 (t )
an (t ) =
= 2 (t ) R,
(1.25)
R
&
a (t ) = (t ) R.
.
,
. -

1.

15

,
.
( ),
.
, , , .
S S
(. 1.4) - (, ) S (, ) O' S'
S':
r (t ) = rO (t ) + r (t ),
(1.26)
(t ) = O (t ) + (t ),

a (t ) = aO (t ) + a(t ).
O a O .
S'
S

r (t )

r (t )

rO' (t )
O'
O
. 1.4. M
S S

, :
f r (r1 , r2 ,..., rN ) = 0,

f (1 , 2 ,..., N ) = 0,
f a (a1 , a2 ,..., a N ) = 0.

(1.27)

16

.
1. . -
,
()
:
ri = rik .
(1.28)
k i

2.
(, , , ..) ,
(, , ), .
1.2.
1.2.1.
,
.

:
1) ,
2) ,
3) ,
4) .
, ,
.
1.2.2.
I. .
1. , .
2. ( ).

1.

17

3.
.
4. (
).
II. .
1. :
) ,
) ,
) .
2. .
3. .
4.
(,
).
III. .
1. .
2. ( , ,
).
3. .
.

. I.3 II.2 , . II.3 .
. I.3 II.2 .
II.1 II.3 ( II.2.a II.2.) .
1.3.
1.1
( )
: = ci + bxj ,

18

c b .
- : r (0) = 0 .
:
) r (t ) , (t ) a (t ) , a (t ) an (t ) ;
) y(x) ;
) (t ) ;
) (t ) (t ) a (t ) .

.
I. XY,
,
i j .
II.
:
x(0) = 0, y (0) = 0,
(1.29)

x (0) = c, y (0) = b 0 = 0,
dx

x (t ) = d t = c,
(1.30)

(t ) = d y = bx(t ).
y
dt
III.
(1.29)
(1.29) -
r (t ) :

x(t ) = ct ,

cbt 2
(
)
=
,
y
t

r (t ) = cti +

cbt 2
j.
2

(1.31)

(1.32)

1.

19

x(t) (1.31), (t ) = ci + bx(t ) j a (t ) :


(t ) = ci + cbtj ,
(1.33)
d
a (t ) =
= cbj .
(1.34)
dt
(1.31) t:
cb x 2 b 2
y ( x) = 2 =
x .
(1.35)
2 c
2c

, . 1 .
(1.7) :

(t ) = x2 + y2 = c 2 + c 2b 2t 2 .

(1.36)

a (t ) an (t ) (1.19, 1.23) :

d
c 2b 2t
cb 2t
,
(
)
=
=
=
a
t

dt
1 + b 2t 2
c 2 + c 2b 2t 2

c 2b 4t 2
cb

2
2
2 2
(
)
a
t
a
a
c
b
=

=
.

2 2
n
1+ b t
1 + b 2t 2

(1.21) :

(c

3/ 2
+ c 2b 2t 2 1 + b 2t 2 c
= 1 + b 2t 2
.
(1.38)
an
cb
b
(t ) (t ) a (t ) :

(t ) =

(1.37)

an
1 + b 2t 2
1
cb
cb
=

= 2 = .
(1.39)
2
2
2
a
cb t
cb t bt
1+ b t
, (1.35) ,
Y (1.34). . 1.5

.
tg (t ) =

20

Y
, t = 0
y (x)
.
a
,

c
0
= , O
X
b
. 1.5
= / 2 .
t ,
, , ,
.

1.2
( )
H 0 . ,
, ,
.

I.
0 (t = 0)
(. 1.6).
,
. X - Y
0
g
H
0 , Y

X
O
. , , . 1.6

g .

1.

21

II.
:
x(0) = 0, y (0) = H ,
(1.40)

x (0) = 0 , y (0) = 0;
d x

a x = d t = 0,
(1.41)

a = d y = g .
y
dt
III.
(t )
r (t ) :
x = 0 ,
(1.42)

y = gt ;
x = 0t ,

(1.43)

gt 2
.
y = H
2


(1.43) t:
gx 2
y ( x) = H 2 .
(1.44)
20

, . 1 .
(1.8) :

= x2 + y2 = 02 + (gt ) 2 .

(1.45)

(1.14) :
a = a x2 + a 2y = g .

(1.46)

(1.19, 1.23) :
d
g 2t
0 g
, an = a 2 at2 =
.
(1.47)
a =
=
dt
02 + g 2t 2
02 + g 2t 2

22

(1.21) :

3/ 2

+ g 2t 2
.
(1.48)
an
0 g
,

t0 = 0 t0 t t.
(1.43),
y :
2H
t =
.
(1.49)
g

2
0

1.3
(
)
, .
, d,
, 0 um.
, ,
l .

I. ,
, .
u( y ) . 1.7.
u(y)

Y
X
. 1.7

, .

1.

23

II.
:
x(0) = 0, y (0) = 0,
(1.50)

x (0) = 0, y (0) = ,

x , y
.
(1.26)
(t ) = u( y (t )) + (t )
:
x = u ( y ),
(1.51)

y = .
,
d, , :
2
u ( y ) = a( y d / 2) + b ,
(1.52)
a b . b
:
b = u ( y = d / 2 ) = um .
(1.53)
x (0) = a

d2
+ b = 0 4

(1.53), a:
4
4
a = 2 b = 2 um .
(1.54)
d
d
III. (1.51) (1.52) (1.54) :
4um 2 4um
d x
d t = x = d 2 y + d y,
(1.55)

d y = = .
y

d t
(1.55)
(1.50), :
3
4u
2u
x(t ) = 2m 2 t + m t 2 ,
(1.56)
3
d
d

24

y (t ) = t .
(1.57)
, t
(1.56) (1.57):
4um 3 3 2
x( y ) =
(1.58)
y + y d .
2
3 d 2

y ( ) = d ,
:
d
= .
(1.59)

, l (. 1.58):
2u
l = x( ) = m d .
(1.60)
3
1.4
( )

, 1 ,
a .
, 2 .

I.

(. 1.8).
X

'

X'

2
Y'

. 1.8

1.

25

XY, , ,
X
, Y . XY, , ,
X Y X Y. .
, , XY.
II. (1.26), a
( XY):
= 2 1 ,
(1.61)

a = a .
(1.62)
:
x(0) = 0 , y(0) = 0;
(1.63)
x (0) = 1 , y (0) = 2 .
(1.64)
III. (1.61) (1.62)
(1.63) (1.64)
:

at 2
,
x = 1t
(1.65)
2

y = t.
2


(1.65) t:
y
y 2
(1.66)
x = 1 a 2 .
2
2 2
, XY,
, (. . 1.9)
:
x' =

12
2a

, y' =

12
a

(1.67)

26

X'

x = x ( y )
Y'
. 1.9

1.5
( )
MN
MON (. . 1.10). P , b.

I. ,
MON (. . 1.10).
Y
M
y

(t)
O

. 1.10


. , t (t) OX
MN.
II. P
(. . 1.10):
x(t ) = a cos (t ),
(1.68)

y (t ) = b sin (t ).

1.

27

P , (1.68).
III. (1.68), :
x2 y 2
cos 2 (t ) + sin 2 (t ) = 2 + 2 .
(1.69)
a
b
,
:
x2 y2
+
= 1.
(1.70)
a 2 b2
(1.70) ,
a b. , a = b,
.
1.6
( )
, , 1 2 (.
. 1.11), ,
, .

, 2
.

I. , . X , Y (. . 1.11).
Y

y
y1
y2
O

1
2

x1
. 1.11

x2

28

1 2 , , , X. XY (x1, y1)
(x2, y2), . ,

.
II. l
:
y y
(1.71)
l = 1 + y y2 ,
sin
y , .
, :
x x
l = 2 1 + y y2 .
(1.72)
cos
III. (1.71) (1.72) , l = const y = const ,
:
a y1 a y 2 sin = 0,
(1.73)

a x 2 a x1 a y 2 cos = 0.
1.7
( )

(. . 1.12). ,
r R. , r, , . ,
, a1. .

1.

29

I. , Y
. - R
r
, , . 1.12.
1 2 X
, .
1

.
II. 2
t
. 1.12
x1 ( , ). , r x1 :
x1 = r .
(1.74)
R
, ,
2, :
l = R .
(1.75)
, , :
l
x2 =
.
(1.76)
2
III. (1.74) (1.76), , :
R x
(1.77)
x 2 = 1 .
r 2
(1.77)
,
:
R
2 = 1 .
(1.78)
2r
, :

30

a2 = a1

R
.
2r

(1.79)

1.8
( )
R,
A
, ,

, (.
R
. 1.13).

:
2
x = x0 + bt , x0 b .

, a,
an a . X
.

. 1.13

I. .
X (. 1.13).
A Y,
A. ,
.
II. :
x = x0 + bt 2 .
(1.80)
,
:
d
x = R
.
(1.81)
dt

1.

31

(1.24) (1.25) , . 1.1.


III. ,
(1.6) (1.12):
dx
x = = 2bt , y = 0 ;
(1.82)
dt
d
ax =
= 2b , a y = 0 .
(1.83)
dt

, ( )
, , (1.22)
, :

2bt
= x =
,
(1.84)
R
R
d 2b
=
=
.
(1.85)
dt
R
X , :
a = 2b .
(1.86)
,
(1.22):
2 4b 2t 2
an =
=
.
(1.87)
R
R
A
(1.20):
4b 2t 4
+1 .
(1.88)
R2
A :
t2
bt 2
(t ) = 0 +
= 0 +
,
(1.89)
2
R
0 A .
a = an2 + a2 = 2b

32

1.9
( )
, ,
: r = r(t), = (t).
Y

er
e
, r
M
, j
.

, X
i
O
X
(.
. 1.14
. 1.14).

I. Y ,
XY , M (. 1.14).
XOY c
i j , , er e . 1.14.

, er e ,
i j .
II, III. , , XOY:
x (t ) = r (t ) cos (t ),
(1.90)
y (t ) = r (t ) sin (t ).
(1.90) ,

:
x = x& = r& cos r& sin ,
(1.91)
y = y& = r& sin + r& cos ;

1.

33

a x = & x = ( &r& r& 2 ) cos ( 2r&& + r&&) sin ,

(1.92)
a y = & y = ( &r& r& 2 ) sin + ( 2r&& + r&&) cos .
(1.92), (1.92) .
.
1 . :
= r er + e ,
(1.93)

a = ar er + a e .

(1.94)

, ,
, :
x = i = r er i + e i = r cos sin ,
(1.95)
y = j = r er j + e j = r sin + cos ;
a x = a i = ar er i + a e i = a r cos a sin ,
a y = a j = ar er j + a e j = ar sin + a cos .

(1.96)

(1.90) (1.95), (1.91)


(1.96), :
r = r&,
(1.97)
= r& ;
a r = &r& r& 2 ,
a = 2r&& + r&&.

(1.98)

2 . - :
r = rer .
(1.99)

e r e ,
(. . 1.15):
e&r = & e ,
(1.100)
e& = & e r .

34

de

de r

e
d

er

X
. 1.15


(1.99) (1.100):
= r& = r&e r + re&r = r&e r + r&e ,
(1.101)
a = & = &r&e r + r&e& r + r&&e + r&&e + r&e& =
= ( &r& r& 2 )e r + ( 2r&& + r&&)e .
(1.102)
(1.101) (1.102) :
r = r&,
(1.103)
= r& ;
a r = &r& r& 2 ,
a = 2 r&& + r&&.

(1.104)

, .
1.10
( )

r ( ) = 2a (1 + cos ) ,
(t ) = bt . .

I. .
, M -

1.

35

, ,
2
( t )
,
b
M
(. . 1.16).
r(t)
II. a(t k )
(t)
X
O
, (1.103)
. 1.16
(1.104), :
r = r& = 2a sin & = 2ab sin ,
(1.105)
= r& = 2a (1 + cos )b;
a r = &r& r& 2 = 2ab 2 ( 2 cos + 1),

(1.106)
a = 2r&& + r&& = 4ab 2 sin .

:

= r2 + 2 = 2ab 2 + 2 cos(bt ) ,

(1.107)

a = ar2 + a2 = 2ab 2 5 + 4 cos(bt ) .

(1.108)

,
tk = (2k + 1)

( k = 0, 1, 2, ...) () b
, , ,
a(tk ) = 2ab 2 , .
1.11
( )

r (1 e cos ) = p .
p , e .
r .

36

I. . , O (. . 1.17).

r&t
r(t+t)

(t)
O

r(t)
X

. 1.17


, , .

, , - , , .
II.
(1.104):
a r = &r& r& 2 ,
(1.109)
a = 2r&& + r&&.
(1.109) , :
r (1 e cos ) = p ,
(1.110)
1
= r 2& .
(1.111)
2
III.
,
:

1.

37

1
(1.112)
2
(1.112) (1.109) a , , a = 0 . ,

& = r& + r 2&& = 0 .

ar , (1.109)
.
(1.110)
:
r&(1 e cos ) + re sin & = 0 .
(1.113)
(1.110)
(1.111), (1.113) :
(1.114)
r& p + 2 e sin = 0 .
(1.114)

&r&p + 2 e cos & = 0 .


(1.115)
(1.110) (1.111) cos &
(1.115):
r p 2
r p
&r&p + 2e
(1.116)
2 = &r&p + 4 2 3 = 0 .
er r
r
:
r p
&r& = 4 2 3 .
(1.117)
r p
a r ,
, &r&
2
(1.116) & = 2 (. (1.111)) (1.109):
r
4 2
4 2
r p
(1.118)
a r = &r& r& 2 = 4 2 3 r 4 = 2 .
r p
r
r p
, , , ,
:

38

4 2
,
r 2p
a = 0.
ar =

(1.119)

1.12
( )
R. 0 . ,
(t ) .

I. , , g .
,
Z ,
. 1.18. er, e ez . ,
, .
Z

ez
O

er
X

. 1.18

1.

39

II.

:
(1.120)
r (0) = 0 , (0) = 0 , z (0) = 0 .
(1.103) (1.104)
, :
r = r& , = r& , z = z& ,
(1.121)
a r = &r& r& 2 , a = 2r&& + r&& , a z = &z& .
(1.122)
, , :
r (t ) = R , a (t ) = 0 , a z (t ) = g .
(1.123)

III. (1.121) (1.123),


:
r (t ) = 0 , (t ) = 0 , z (t ) = gt ;
(1.124)
ar (t ) =

02

, a (t ) = 0 , a z (t ) = g .
(1.125)
R
, :
= 0e gte z ,
(1.126)
a=

02

e r ge z .
(1.127)
R
:
a
g 2t
cos =
.
(1.128)
=
a
2 2

02 + (gt )2 0 + g 2
R

1.4.
1
, ,
, -

40

. . , :
1) y ( x ) ;
2)
y ( x ) ;
3)
y ( x ) .
g
g
: 1) y =
x 2 , 2) y = 2 ( x) 2 ,
2
2
2( + )
3) y =

( x) 2 , x < 0 . X, X' X'' -

2
, Y,
Y' Y'' ,
XY .
2

2
d , ,
, 0 u. ,
l .

: y =

d
u

x , y < d/2; y = d

d 2 d

x , y > d/2;
2
u

ud
. X XY
2
, Y . ,
,
.
l=

3

.
u. N
. N' ,
?

1.

: N =

+u

41

N.

4
, 1 2 ,
(. .).

2

, 1 2 , .
: =

12 + 22 21 2 cos
,
sin

, 1
2 + 2 2 cos
2
1 2
1

1 sin

1 = arccos
1 .

5
,
, = (. .).
L

. L. A

42

, H
.
2

L
: A = 1 +
.
2H
6
, . R << H.
.
L > . .
RL
( rH ) 2
: =
, a=
.
32
L2 H 2
L2 H 2

7.

,
(. .).
: a1 + 2a2 + a3 = 0 , a1 , a 2 a 3

.

3
2

8

, ,
: r (t ) = a (1 bt ) ,
bt
(t ) =
, a b .
1 bt

: = ab 1 +

1
1
, t < .
2
b
(1 bt )

1.

43

9
(. .).
4,
.
: a4 = (a1 + a2 + 2a3 ) 4 , a1 , a2 ,
a3 a4 .

4
3

10
,
, .

r
R
1

1
1
R+r
r
a2 + 2a3 + a4 + a5 = 0 ,
2
2
R
R
a1 , a2 , a3 , a4 a5
.
: a1 +

44

11
, .
2, 1.
: a2 = 16a1 , a1 a2
.

2.

45

2


2.1.
2.1.1.
. , (
) . .
.

, :
ma = Fi .
(2.1)
i

. :
1) ,
2) ,
3) ,
4) ,
5) ,
.
,

:

ma x = Fix ,
i

(2.2)
ma = Fi ma y = Fiy ,
i
i

ma = F .
iz
z
i
, , , .

46


(2) (),
. , ,
n :
man = Fin ,
i

(2.3)

ma = Fi ,
i

an (t ) = an (t )n(t ) a (t ) = a (t ) (t ) .
, .
2.1.2. ,
.
. F21 F12 (. . 2.1),
:
mm
F21 = F12 = G 1 3 2 r12 .
(2.4)
r12
G = 6,6731011 2/2 ,
r12 = r2 r1 .
S

r1

m1

F21 F12 r12 m2


r2

. 2.1.

, ,

2.

47

(2.4), r12 - .
, ,
( ) (.
4), ,
.
, , ,
.

F m
(
) g : F = mg .
, , ,
, .
.
, .
. .
, ()
(, ) ,
:
1
l = F .
(2.5)
k
k () , l = l l0
, l l0
(. . 2.2).
, , . 2 , . l < 0 F (2.5) F X , . 2.2.

48

l0

d0

. 2.2.

()
:
1
= .
(2.6)
E
E , ,
F
l
=
, =

S
l0
, S .
, ()
:
S
k = E.
(2.7)
L
() () . :

(2.8)

d d 0 d
=
d0
d0
, d d0
(. . 2.2).

, =

2.

49

F , , .
F
= .
(2.9)
S
x x + dx
(. . 2.3).

(x)

(x+dx)

(x)

(x+dx)

x+dx

. 2.3.


(x) (x+dx) .

( x + dx) ( x)
=
=
= x' .
(2.10)
dx
x

( x) = E = E x' .
(2.11)
.
(2.8).

, ,
. - (2.11), (2.8).

50

.
.
( ) .
() F , ( )
.

F = ,
(2.12)
() .
: F = 0 .
Fc ,
.
F ,
.
Fc , .
, :
1) :
0 F Fmax ;
2) : Fc = Fmax ;
3)
:
Fc = N ,
(2.13)
( ) ,
, ;

2.

51

4) :
Fc .
(2.14)
,
, ,

F (t , r , ) .
2.2.
2.2.1.

, , .

,
.
,
. ,
,
.
2.2.2.

I. .
1. , .
2. ( ).
3. .
4. (
).
II. .
1. .

52

2. , . 3.
3. ,
:
) ,
) ,
) ..
4. .
5.
.
III. .
1. .
2. ( , , ).
3. .
.

. I.3 II.1 ,
. II.2 .
. I.3 II.2 , .
II.1 II.4 ( II.3. II.3.) .
2.3.
2.1
( )
, . m1 m2. .

2.

53

( ) .
I. , . 2.4,
: , .
. ,
, . , ,
,
.
II.
X (.
T
. 2.4)
, :
T
m1a1 = m1 g T1 ,
T1 T2
m2 a2 = m2 g T2 ,
(2.15)
T1
mg
a1 + a2 = 0.
T2
a1 a2
T
X, T1 T2 ,
m1g
X
m2g
.
. 2.4
. (. . 2.4) X:
m a = T T + m g + F ,
(2.16)

m , a X, T T ,
, F .

54

, (2.16) , , , , ,
.

(. (6.30) 6) (. . 2.4):
d
J
= T1R + T2 R + M + M ,
(2.17)
dt
J
,
, , M , ,

M , .
,
, (2.17)
. ,
, :
T1 = T2 .
(2.18)
III. (2.15) (2.17) :
m m2
m m2
a1 = g 1
, a2 = g 1
.
(2.19)
m1 + m2
m1 + m2
. , , , . m1 >> m2, a1 = g, a2 = g. m1 << m2, a1 = g a2 = g.
. ,
, .
.

2.

55

2.2

, .

I.
, . 2.5, Y
, .
T1
T1
,
, T1
: T2
,
T2
,
m1g
. , X
m2g
,
. 2.5
,
.
II. X
:
m1a1 = m1 g T1 ,
(2.20)
m2 a2 = m2 g T2 .
(2.21)
a1 a2 X, T1 T2 , .
x1, x2 x
. (.
. 2.5) :
x1 + 2 x = const ,
(2.22)
x2 x = const .
(2.23)
(2.22) (2.23) ,
:
a1 + 2a2 = 0 .
(2.24)
(. 1).

:

56

m a = T2 2T1 ,
(2.25)
m = 0 .
III. (2.20), (2.21), (2.24)
(2.25), :
2m m2
a1 = 2 g 1
,
(2.26)
4m1 + m2
a2 = g

m2 2m1
,
4m1 + m2

T1 = m1 (g a1 ) =

3m1m2 g
,
4m1 + m2

(2.27)
(2.28)

6m1m2 g
.
(2.29)
4m1 + m2

(2.26) (2.29).
1. 2m1 = m2 , :
a1 = a2 = 0 .
:
T2 = 2T1 = 2m1 g .
2. m2 >> m1 , m2 :
a2 = g ,
m1 :
a1 = 2 g .
:
T2 = 2T1 = 6m1 g .
3. m1 >> m2 , m1 :
a1 = g ,
m2
a 2 = 0,5g .
:

T2 = 2m1 (g a1 ) =

2.

T2 = 2T1 =

57

3
m1 g .
2

2.3
, . 2.6,
m M, .
, , .
A .
Y
T
N

y0
y

R
mg
N

Mg

. 2.6

I. , . 2.6.
, : mg Mg
, , ; R
, ; N
. , T ,
- .
II.
,
X ( Ay = 0 ):

ma x = N sin T cos ,

(2.30)

58

ma y = N cos mg + T sin ,

(2.31)

MAx = N sin + T cos T ,


(2.32)
0 = R N cos Mg T sin .
(2.33)
(2.30) (2.33) ,
:
x x
x +
= const ,
(2.34)
cos
y y
(2.35)
x + 0
= const .
sin
(2.34) (2.35) , :
a Ax
= 0,
(2.36)
Ax + x
cos
ay
(2.37)
Ax
=0.
sin
III. (2.30) (2.32), (2.36),
(2.37), ax a y . (2.33) (
,
R). :
mAx (1 cos ) = N sin T cos ,
(2.38)
mAx sin = N cos mg + T sin ,
(2.39)
MAx = N sin + T cos T .
(2.40)
, X:
sin
Ax = g
.
(2.41)
M / m + 2(1 cos )
Ax . (M >> m), . .
, ,
.

2.

59

2.4
m.
.
.
F, F = t, = const. (t)
V(t) t .

I. .
F
, . t 0
, ,
, ,
, . ,
:
0 t t0 t > t0 . t0 , .
, . 2.7,
, .
Y
R
N
F

F
Mg

mg

N
. 2.7

II.
,
, ( t t0 )

60

( t > t0 ). : a A X.


ma = F F ,
(2.42)
0 = N mg ,
MA = F ,

(2.43)
(2.44)

0 = R N Mg .
(2.45)

F :
(2.46)
F = t .
,
- , .
,
:
:
(2.47)
a = A.
(2.48)
F = N .
III.
.
(2.45) , R,
.

t
,
M +m
Mt
F =
.
M +m

a= A=

(2.49)
(2.50)

a=

t mg

m
mg
A=
,
M
F = mg .

(2.51)
(2.52)
(2.53)


(.
(2.13)):
Mt0
= mg .
(2.54)
M +m

2.

61

(2.54) t0 ,
:
mg (M + m )
.
(2.55)
t0 =
M
:
t

t t0 : a = A = M + m ,
(2.56)

t > t : a = t mg A = mg .
0

m
M
, .
t t0
t :
t

=V = adt =
0

t
M +m

dt =

t 2

2(M + m )

(2.57)

t > t0
t

t02

2(M + m )

t02

V=

t02

2(M + m )

t0

t 2 t02
2m
t

2(M + m )

t mg

+
t0

mg
M

dt =,

) g (t t ) ,

dt =

t02

2(M + m )

,V

mg
M

(t t0 ) .

,V
0

(2.58)

V
t0

. 2.8

(2.59)

62

. 2.8
.
2.5
( )
,
0,
F.

I. , Y
. 2.9,
0
F

II. O
X

. 2.9
:
d
m x = 0,
(2.60)
dt
d y
m
=F.
(2.61)
dt
III. (2.60) (2.61), x (0) = 0 sin y (0) = 0 cos :

x (t ) = 0 sin ,

(2.62)

F
t.
(2.63)
m
(2.62) (2.63)
x0 = 0 y0 = 0, :
x(t ) = 0 sin t ,
(2.64)

y (t ) = 0 cos +

Ft 2
.
(2.65)
2m
(2.64) (2.65), :
y (t ) = 0 cos t +

2.

63

F
x 2 + ctg x
(2.66)
2m( 0 sin ) 2
,
.
y=

2.6
.
0 0
(. . 2.10).

0
0

. 2.10

= tg .
.

I. , . 2.11
( ) . 2.12 ( ). X

(. 2.12). Y , Z
(. 2.11).
N
F

mg

(t)
mgsin

(t)

Y
. 2.11

. 2.12

64

. 2.11 2.12 ,
: mg, N F.
(t) X (t) (.
. 2.12), .
II. :
d
m x = F cos ,
(2.67)
dt
d y
m
= mg sin F sin ,
(2.68)
dt
0 = N + mg cos .
(2.69)
(. . 2.1.2.)

:
F = N = tg N .

(2.70)

,
:

y
x
, sin =
.
(2.71)

III. (2.67) (2.71)



, -

.
. , .. ,
, , .
. : . (2.69) (2.70) :
F = mg sin .
(2.77)
,
. cos =

2.

65

, Y. , ,
. , Y.
, ,
Y . (2.67) (2.71)
:
d x

= x g sin ,
(2.72)
dt

d y y
= 1 g sin .
(2.73)
dt

:

d d
d d
=
x2 + y2 = x x + y y .
dt
dt dt
dt
(2.72) (2.73) (2.74) :

(2.74)

d y
d
=
.
(2.75)
dt
dt
(2.75) ( (0) = 0 ,
(0) = 0 ), :
= y + 0 (1 + sin 0 ) .
(2.76)
= y = (2.76), :

(1 + sin 0 ) .
(2.78)
2

.
0 = /2 (
), = 0. ,
, .

66

0 = /2
= 0. ,
, .
,
.
.
. , ..
.
2.7
r , , . (t), . , 1, 2. ,
: F = 6r .

I. , , ,
. 2.13. Y
F
. - F

: (0) = 0 .
II. mg
X :
ma = mg F FA ,
(2.79)
X
F , FA
. 2.13
.
, :
FA = 1 gV ,
(2.80)
F = 6r .
(2.81)

2.

67

4
V = r 3 .
3
:
m = 2V .
(2.82)
III. (2.80) (2.82) (2.79),
:
d
2V
= ( 2 1 )Vg 6r .
(2.83)
dt
(2.83)
d ( 2 1 )
6r
=
g
= A B
(2.84)
2
2V
dt
:
(2.85)
A B = z .
(2.85) , :
d d z
.
(2.86)
B
=
dt dt
(2.86) (2.84) :
dz
= Bz .
(2.87)
dt
(0) = 0 :

z = Ae Bt .
(2.88)
(2.85), :
A
= 1 e Bt .
(2.89)
B
A B (2.84), V, :

2r 2
9
(2.90)
1 exp 2 t .
= ( 2 1 )g

9
2r 2
2r 2 2
t >>

9

2r 2
= ( 2 1 )g
.
(2.91)
9

68

2.8

0 ,
(. 2.14). ,
.
R, .
R.

I. ,
n
.
, N
0
,
F
, , n , . 2.14

(. 1).
II. , , :
d
ma = m
= F ,
(2.95)
dt

man = m

=N.
(2.96)
R

:
(2.97)
F = N .
III (2.95) (2.97) :
2
d
=
.
(2.98)
dt
R
(2.98) , :

d
= dt ,
(2.99)
2
R

2.

t +C.
(2.100)
R
(2.100)
1
( (0) = 0 ): C = .

69

, t,
,
:
1
.
(2.101)
= 0
0
1+
t
R
, t ,
, 0, > 0.
, ,
, , .
, d t t,
, :
1
(2.102)
d s = d t = 0
dt .
0
1+
t
R
, t
(2.102) :
R
(2.103)
s = ln1 + 0 t .
R

, , (2.103) :

R R s
t=
e 1 .
(2.104)

s = R
t0 :
R
t0 =
e 1 .
(2.105)

70

( << 1 )
t0
R
R
((1 + + ...) 1) R .
t0 =
e 1 =
(2.106)

2.9
= 1 , m = 2 . F ,
- ?
1 = 0,25, 2 = 0,5.

I. , . 2.15
, .
Y
R

N
F1

F1

F2
N

mg

Mg

. 2.15

mg, F1 N. F, Mg,
R, N
F1 F 2 . .
. F , ( ), F1
.

2.

71

F F1

F0 ,
. F0
.
II.
:
ma = F1 ,
(2.107)
0 = N mg ,
MA = F F1 F 2 ,

(2.108)
(2.109)

0 = R N Mg .
(2.110)
a A X.
, :
F 2 = 2 R .
(2.111)
F0 , , , :
a = A,
(2.112)
F1 = 1 N .
(2.113)
III. (2.107) (2.113)
F0 :
F0 = (M + m )a + F2 = (M + m )g (1 + 2 ) .
(2.114)
, , -
, F,
:
F F0 = (M + m )g (1 + 2 ) .
(2.115)
, ,
:
F F0 = 22,5 .
(2.116)
2.10
( )
F, m XY
.

72

x (t ) = A sin(t ) , y (t ) = B cos(t ) , A, B, .

I. ,
.
II.
:
d2 x
ma x = m 2 = Fx ,
(2.117)
dt
d2 y
(2.118)
ma y = m 2 = Fy .
dt
:
x(t ) = A sin (t ) ,
(2.119)
y (t ) = B cos(t ) .
(2.120)
III. (2.119) (2.120) , :
d2 x
= A 2 sin (t ) ,
(2.121)
2
dt
d2 y
(2.122)
= B 2 cos(t ) .
2
dt
(2.121) (2.122) (2.117) (2.118), F,
XY :
Fx (t ) = mA 2 sin ( t) ,
(2.123)
Fy (t ) = mB 2 cos( t) .

(2.124)

(2.123) (2.124),
F:
F = Fx2 + Fy2 = m 2 A2 sin 2 (t ) + B 2 cos 2 (t ) .

(2.125)

:
F (t ) = Fx (t ) i + Fy (t ) j = mA 2 sin (t )i mB 2 cos(t ) j =
= m 2 r (t ) ,

(2.126)

2.

73

r - .
, F , , - .
2.11
F0 , .

l0 S 0 , E,
.
(x) (x) x
, .

I. X (.
. 2.16), .
F0
,
, . , , , (x)
(2.11).

(x)

(x+dx)

(x)

(x+dx)

x+dx

. 2.16

II. dx x (. . 2.16). dm
m
dm = dx ,
(2.127)
l0

74

m .

X :
dma = S ( x + dx + ( x + dx)) ( x + dx + ( x + dx))

dx .
(2.128)
S ( x + ( x)) ( x + ( x)) S ( x)
x
a X, (x) (x+dx)
(. .2.16); S ( x + ( x )) S ( x + dx + ( x + dx)) . ,
(2.128) dx,

.
( x) =
x
(2.128)
:
( x = 0) = 0 .
(2.129)
a,
, :
F
a= 0 .
(2.130)
m

, ,
S(x):
S ( x) = S 0 (1 ( x)) 2 S 0 (1 2 ( x)) .
(2.131)
:
( x ) = E ( x ) .
(2.132)
III. (2.127),
(2.128) (2.130) (2.132),
:
( x) ( x)
F0

= S 0 1 2
.
(2.133)

l0
E x

2.

75

(2.133) (2.129), :
F0 x
S
= S 0 ( x) 0 ( x) 2 .
(2.134)
l0
E
(2.134) ( x) :

( x) =

4 F0 x
E
1
1

.
2
S 0l0 E

(2.135)



(2.135):
Fx
( x) = 0 .
(2.136)
S 0 l0
,
:
Fx
( x) = 0 .
(2.137)
S 0 l0 E
,
(2.137):
l0

0
Fl

l = (l0 ) =
dx = ( x)dx = 0 0 .
2S0 E
x
0
0

:
l
F
= 0 .
l0 2 S 0 E

(2.138)

(2.139)

2.4.
1
= 30 m1 , ,
, m1
.
m2

m2 , (. .). -

76

= m2 m1 = 2 / 3 .
= 0.1. .
:
a2 = g ( sin cos ) ( + 1) = 0.05 g .
2
, , 1 2 (. .). m1
m2, 1 2,
2
1 > 2.
1
,

, ,
.
( 2 )m1m2 g cos ; > arctg 1m1 + 2 m2 .
: F = 1
m1 + m2
m1 + m2
3
m R.
, 0 .

: F = m02 R cos 2 .
4
X x = t 2 t 3 ,
. t = 0 ,
, F0 .
, x = 0 .
: F0 , 2F0 .

2.

77

5

M . m
.
. ,
N R, .
:
g (sin cos )
mg (sin cos )
, aM =
,
am =
1
m
M
1
sin (tg )
+

+ sin (tg )
cos M
m cos

mg
(
)
cos
sin
m
+

, R=
N=
+ M g .
1
m
1 + m sin (tg )

+
sin (tg )

cos M
cos M

6
.
M, m a . ,
.
: F = (2 g a)mM (m + M ) .
7
, h,
0 . , .
h( 0 )
: t =
.
0
0 ln

8
, , l.
, l

78

. ,
, 2. ? .
2l
.
: t1 = t2 =
3
9
,
, .
, .
, , (.
.).
m1

m3

m2

m1 m2, m3,
. .
2m1m2
: m3 =
.
m1 + m2
10

, R = 200 = 100 /?

: =

2
Rg

0.4 .

2.

79

11.
m
.
l0 S0 ,
E, . ,
.
l
mg
V (1 2 )mg
:
=
;
=
.
2 ES0
l0 2 ES 0 V0

80

3


3.1.
(
) , - r

r =

mi ri
i

,
(3.1)
m
m = mi , ri mi i

i- .
,

mi i

.
(3.2)
m
i i- .
a ,

mi ai

a =

.
m
ai i- .

(3.3)

3.1.1.
( ) p
,
:
p = m .
(3.4)
P ,
,
:
P = pi = mi i = m p .
(3.5)
i

3.

81


( ) F ex ,
, .
(.
2):
(3.6)
mi ai = ma = F j = F jex + F jin = F ex .
i

F jex

, j-

, F jin = Fijin
i j

, j- , .
F
dt, , , : F d t .

dt ,
:
(3.7)
d P = F ex dt .

t2

P P (t2 ) P (t1 ) = F ex dt ,

(3.8)

t1

t1 t2 .
( n )
, :
d Pn = Fnex dt ,
(3.10)

82
t2

Pn Pn (t2 ) Pn (t1 ) = Fnex dt .

(3.11)

t1

, : F jex = 0 .
,
:
F jex = F ex = 0 .
j


, :
P P (t2 ) P (t1 ) = 0 .
(3.12)

, n : Fnex = 0 .
,

:
Pn Pn (t 2 ) Pn (t1 ) = 0 .
(3.13)
.
. M(t)
t dm = dM dt (. . 3.1).
S

t + dt

t
M(t)

dm

(t )

1 (t )

M(t) dm

( t ) + d

. 3.1.
t t + dt

3.

83

t
t + dt:
P (t ) = M (t )(t ) ,
(3.14)
P (t + d t ) = (M (t ) d m )((t ) + d ) + d m1 (t ) .
(3.15)
(t ) t, d
dt, 1 .

,
dt ,
P (t + d t ) P (t ) = M (t ) d + d m u(t ) ,
(3.16)
u(t ) 1 (t ) (t ) .
, M(t), .. :
d P (t )
d dm
= M (t )
+
u(t ) = F ex ,
(3.17)
dt
dt dt
dm
M (t )a (t ) = F ex
u(t ) = F ex u(t ) = F ex + F (t ) .
(3.18)
dt
dm
dM
F ex , , =

=
dt
dt
, , , F (t ) u(t ) ,
.
3.1.2.
F d r
, ( ), :
A = F dr .
(3.19)
F
, , :

84
r2

A12 = F d r .

(3.20)

r1

r1 r2 - .
,
,
, ( ).
, ,
:
A
N=
= F .
(3.21)
dt
.
F p ,
,
.
1.
, ,
F p,in F p, ex .
, , , :
r
F (r ) = F (r ) ,
(3.22)
r
r - , r = r .
. .
1. .
S O
(. . 3.2), (3.21) 1

, , , .

3.

85


:
r
d A = F (r ) d r = F (r ) d r = F (r ) d r .
(3.23)
r
,
r
r
r
xd x + yd y + zd z r dr
=
. (3.24)
dr = d x+ d y + d z =
x
y
z
r
x2 + y2 + z 2
Z

F (r ) = F (r)
r

dr

r
r

dr

. 3.2.
S O


r2

A12 = F (r ) d r = f (r2 ) f (r1 ) ,

(3.25)

r1

f (r ) F (r ) .
,
.
2. .
, F1 F2 , ,
, (. . 3.3)
F1 = F2 ,

r
r12 = r2 r1 , F12 = F21 = F ( r12 ) 12
(3.26)
r12

86


:
d A = A12 + A21 = F12 d r2 + F21 d r1 = F12 (d r2 d r1 ) =
r
= F12 d r12 = F (r12 ) 12 d r12 = F (r12 ) d r12 .
(3.27)
r12
Z

F12 ( r12 )
r12 (t )
F21 ( r12 )

. 3.3.

(
):
2

A12 = F d r = F (r2 r1 ) .

(3.28)

.
, ,
, ,
.
.
F np , , .
(. . 2.1.2. 2) .
, , .

3.

87

, , 2:
dA = F1 dr1 + F2 dr2 =
= (F1 + F2 ) dr1 = (F1 F1 ) dr1 = 0 .
(3.29)
(F2 = F1 )
dr2 = dr1 .
:
dA = F1 dr1 + F2 dr2 = F1 dr1 F1 dr2 =
= F1 (dr1 dr2 ) = F1 dt < 0 ,
(3.30)
.
(3.30) (F2 = F1 )
(. (2.14) 2).
3.1.3.
E p , , ,
( 1)
( 0), :
d E p = Fi p d ri = d Ap ,
(3.31)
i

E p = E1p E0p = d E p = d Ap = A1p0 .

(3.32)


,

( ) E p, in ( , ) E p,ex :

,
.

88

d E p = Fi p,in d ri + Fi p,ex d ri = d Ap,in + d Ap,ex =


i

i
= d E p,in + d E p,ex ,
(3.33)
p
p,in
p,ex
p,in
p,ex
E1 = E1 + E1 = A10 + A10 .
(3.34)

, :
m 2
Ek =
.
(3.35)
2
, :
m2
E k = Eik = i i .
(3.36)
2
i
i

:
E = Ek + Ep .
(3.37)

Fi np,in Fi np,ex 3 :

d E = Fi np,in d ri + Fi np,ex d ri =
i
i

np,in
np,ex
np
= A
+ A
= A ,
(3.38)

E = Anp .
(3.39)
""
.

,

,
(3.34).

3.

89

:4
E E (t2 ) E (t1 ) = 0 E (t1 ) = E (t2 ) .
(3.40)

(3.39).
, .
3.1.4.
()
,

.
,

.
, .
3.2.
3.2.1.

( )

:
1) ( ) ,
2) ( ) ,
3) ( ),
4) ( ),
5) (
).

, , -

90

3.2.2.
I. .
1. , .
2. ( ).
3. .
4. ( ), ,
.
5. (
).
6. ( ) .
II. .
1.

.
2. .
3.
.
III. .
1. .
2. ( , ,
).
3. .

.
I.6 II.2 .

, .

3.

91

3.3.
3.1
( )
= 45 . ,
.
0 = 2,2 /, k = 10 .


.
I. , . X , Y .
. +
X
, ,
, , , .
II. (3.13)
X
:
(3.41)
m + m x = 0 .
x X.
X x
0 :
x = + 0 cos .
(3.42)

:
m
=k.
(3.43)
m

92

III. (3.41) (3.43), X :


m
1
= 0 cos
= 0 cos
.
(3.44)
m + m
1+ k
(3.44) ,
,
0,14 / .
(3.45)
3.2
( )
,
,
. ,
, . , V. V1 V2 ,
, m .

I.
. , ,

.
. 3.4 : t1 , t2 ,
, t 3 .
. 3.4
XY, ,
.
(t1, t2) : + + . ,
,
X (

3.

93

; . . 3.4)
.
X
Y

V12x

V1

V12x
V12y
V22x
V22x

V2
t = t1

V22y
V
t = t2

t = t3

. 3.4

(t2, t3) : + , + .

( ) X,
,

.
II. .
+ ,
(t1 , t2 ) :
( M + m)V1 = MV12 x + mV12 x .
(3.46)
(3.46) ,
:
V12 x = V1 .
(3.47)
,

+ :

94

V 22 x = V 2 .
(3.48)
+ ,
(t 2 , t 3 ) :
MV1 mV2 = 0 .
(3.49)
(3.49) , .
+
:
mV1 MV2 = (m + M )V .
(3.50)
III. (3.49) (3.50), :
m
M
V1 = V
, V2 = V
.
(3.51)
M m
M m
.
, (3.51) . , M = m (V = 0).
,
: V1 = V2 .
3.3
( )

0 = 1 / ( ) M0 = 100 .
t0 = 0 ,
m = kt, k = 10 /.
, . .
?
L = 9 ?

I.
.
+
dt -

3.

95

, , . ,
. 3.5.
II. X + dt [t, t + dt] :
M (t ) (t ) + d m 0 = (M (t ) + d m )( (t ) + d ) .
(3.52)
M(t) (t ) t; dm
dt, d .

. 3.5

,
(3.53)
dm = kdt .
, , (3.52) :
0 = M (t )d (t ) + (t )dm .
(3.54)
:
d
k dt
.
(3.55)
=
(t )
M (t )
.
1.
(M(t) = M0), ,
(3.55), :
k

ln 1 =
t.
(3.56)
M0
0
,
:

96

1 (t ) = 0e

k
t
M0

.
(3.57)
, (3.57):
k
k
t
t

t
M 00
M0
M0
x1 (t ) = 1 (t ) d t = 0 e
dt =
1 e
.
(3.58)

k
0
0

(3.58), ,
L = 9 :
M
1
t1 = 0 ln
(3.59)
23,0 .
Lk
k
1
M 00
2. :
M (t ) = M 0 + kt ,
(3.60)
(3.55) :
k dt
d2
=
.
(3.61)
2 (t )
M 0 + kt
(3.61) :

M0
ln 2 = ln
.
(3.62)
0
M 0 + kt
:
M0
2 = 0
.
(3.63)
M 0 + kt
(3.63) :
t
t
M 00
M
kt
.
x2 = 2 (t ) d t =
d t = 0 0 ln1 +
(3.64)
+
M
kt
k
M
0
0

0
0

(3.64), ,
L = 9 :
Lk

M
t2 = 0 e M 0 0 1 14,6 .
(3.65)

III. . ,
,

3.

97

. . 3.6.
x,
2
150
x1max
1
100
L
50

0
0

100

200
t,

300

400

. 3.6

(3.58) M
x1 max = 0 0 = 10 . k
, .
3.4
( )

. ( ) M0 = 105 ,
u = 2600 /.
(t) .

I. ,
, X .

+ . .
II. (. (3.7))
M (t ) 0 d m u = M (t ) g d t .
(3.66)
M(t) t;

98

dm = (t )dt
(3.67)
dt.

:
(3.68)
dM +dm = 0.
III. (3.66) (3.68) (t ) .
M m,
(t ) :
u
d (t ) = (t )dt .
(3.69)
g
(3.69) :
g
ln (t ) = t + const ,
(3.70)
u

(t ) =

g
t
u
Ae

.
(3.71)
A (3.73) (3.66), :
M g
(3.72)
A = (0) = 0 .
u
:
g

M0g u t
e .
(3.73)
u
, t, (t ) :

(t ) =

g
t

t
m(t ) = (t )dt = M 0 1 e u .
(3.74)

u
t <<
g
:
g
m(t ) M 0 t .
(3.75)
u

3.

99

u 2600

c 260 c , g
9,8
(3.75) :
g
(3.76)
m(t ) t =1 c M 0 t
384.6 .
u t =1 c

3.5
( )
m, l0, .
F, .
lmax. k.

F (. . 3.7), .

x10

x20

X
F

x11

x21

. 3.7

,
, X ,
x10, x20 x11, x21 ( . 3.7). l0 = x20 x10 ,
lmax = x21 x11 .
, , F - -

100

. ,
lmax, , .
,
.
(. . 3.1).
II. (3.39)
+
:
m 2 k (lmax l0 ) 2
( E k + E p ) = 2
+
= A,
(3.77)
2
2

,
A = F ( x21 x20 ) .
(3.78)
(3.6) + :
2 ma = F .
(3.79)

a ,
x11 + x21 x10 + x20

=
,
2
2
2a
2

(3.80)

= .
III. (3.79) (3.80),
:
Fx +x
x + x20
2
(3.81)

= 11 21 10
.
m
2
2

(3.77), (3.78) (3.81), :


F
F
k=
=
.
(3.82)
( x21 x11 ) ( x20 x10 ) lmax l0

3.

101

, , F , lmin, F
.
k =
l0 lmin
, +
(. 4.1 4).
3.6
( )

R
. 0 , . 3.8. ,
.

I. , , .
Y

mg

. 3.8

, N, , .
.

102

,
. 3.8. ,
.
II.
(. (3.39)) :
E2p + E2k E1p + E1k = A ,
(3.83)

) (

m 2 m02

2
2
, A
.
N, .
dt :
A = N d t .
(3.84)
, N , .

n (. 3.8):
E2p E1p = mgR E2k E1k =

= N mg cos ,
(3.85)
R
d
m = mg sin ,
(3.86)
dt
m ,
, n .
, ,
R, , (. 3.8):
d
= R
.
(3.87)
dt
(.
(1.26) 1,) :
= 0 + .
(3.88)
m

3.

103

III. (3.83) (3.88)


.
(3.85) (3.88) (3.84):
gR cos
sin d .
(3.89)
A = 3m0
2
A , (3.89)

0:

A = m 2 gR0 .
(3.90)
(3.90)
A, (3.83) :
m 2 m02
mgR +

= m 2 gR0 .
(3.91)
2
2
(3.91)
:
= 0 + 2 gR .
(3.92)
,
(3.40)
, ,
N :
2
m
mgR = 0 .
(3.93)
2
,

= 2 gR .
(3.94)
(. (3.88)), :
= 0 + = 0 + 2 gR ,
(3.95)

(3.92).
, , .

104

3.7
( .)
m1 m2,
, l0
1 2 , ,
, . 3.9.

2 m2

m1
l0

1
X

. 3.9

lmax .

I. ,
, ,
.
mm
F = G 1 2 2 l
,
l = lmax . X
.
II. (3.13) ( . 3.9) ( ) , :
0 (m1 (1 ) + m2 ( 2 ) ) = 0 ,
(3.96)
1 , 2
X.
(3.40) :

3.

105

E k + E p = 0 ,

(3.97)

m ( ) 2 m ( ) 2
E k = 0 1 1 + 2 2 ,
(3.98)

2
2


(. . 3.1)

E p =

l max

l0

1
1
m1m2
.
d l = Gm1m2
2
l
l0 lmax

(3.99)

III. (3.96) (3.99), lmax :


l0
.
(3.100)
lmax =
2
l0 (1 + 2 )
1
G 2(m1 + m2 )
lmax > 0, (3.100)

2(m1 + m2 )
l0 < G
.
(3.101)
(1 + 2 )2
.
3.8

1 2 ,
. 3.10. l.

?

D
C

A
B

1
. 3.10

I. A, B, C D (. . 3.10) , , , .

106

: , X
,
. 3.11, , , . X'
, . 3.12, X.
, , , , ,
, , .

C
A

C
X

A
B

B
. 3.11

X'
u
. 3.12

, , .
II. 1 2, ,
(. 3)

= 1 2 ,
(3.102)
2
uA , uB , uC uD :
+
uA, B = 1 = 1 2 = u ,
(3.103)
2
+
uC, D = 2 = 1 2 = u .
(3.104)
2
,
(. . 3.12).
, A C
,

3.

107

. A
C (" ", " ") :
muA + muC = mu A + muC ,
(3.105)
muA2 muC2 muA2 muC2
+
=
+
,
(3.106)
2
2
2
2
uA uC A C , m
.
B D

:
uB = u , uD = u .
(3.107)
III. (3.103) (3.106)
A C :
uA = u , uC = u .
(3.108)
. 3.13 . 3.14
(3.103), (3.104), (3.107) (3.108).
u
u
A
B

D
C
u
u
. 3.13

u
u
X'

D
C
u

A
B

X'

u
. 3.14

, A C
,
, . (. . 3.15).

108

u
u
D
A

u
u

B
u

B
u

X'
u

. 3.15

X'

. 3.16

uD uB , D B
, , (3.105), (3.106), :
uD = u , uB = u .
(3.109)
A C

(. . 3.16):
uA = u , uC = u .
(3.110)
,
, , ,
. . 3.17 ,
.

X'

u
A

A
. 3.17

1
. 3.18

D
A

. 3.19

1 2
:
1 = u + = 1 ,
(3.111)
2 = u + = 2 .
(3.112)

3.

109

, , 1
2 , ,
, . ,
, ,
1 2 (. 3.18 3.19).
3.9
( )
m1 p1 m2 . p1 p2
,

.

X ,
(.
. 3.20).

p2
p1

p1

Y
. 3.20

, ,
(3.12)
(3.40).

110

II.
(" ", "
"):
p1 + p2 p1 = 0 ,
(3.113)
p12
p 2
p2
(3.114)
+ 2 1 =0.
2m1 2m2 2m1
(3.114)
p2
E k =
.
2m
III. (3.113)
(3.114)
( p1 p2 ) = p1 p2 cos ,
(3.115)
:
4m1m2
(3.116)
p1 = p1 1
cos 2 ,
(m1 + m2 ) 2
2m2
p1 cos .
(3.117)
(m1 + m2 )
X
(. 3.20). (3.113)
Y:
p1 sin p2 sin = 0 .
(3.119)
(3.119) (3.116) (3.117) :
p

= arcsin 2 sin =

p1

p2 =

m2 sin 2
= arcsin
(m + m ) 2 4m m cos 2
1
2
1 2

(3.120)

3.10
M
,
, N m << M .

3.

111

.
, ? .

I. X , ,

(.
1
. 3.21).
, ,
0
. ,
X
.
. 3.21
II.

:
2
2
m 0
m1
= mgH +
.
(3.121)
2
2
0 1 , H
.
X
p = 2m1 .
(3.122)
t0 N . , N , t0 F, :
pN = Ft0 .
(3.123)
:
Mg F = 0 .
(3.124)

112

g, :
t
0 = 1 + g 0 .
(3.125)
2
III. (3.121) (3.125)
H :
2 (mN + 2M )mN
H= 0
.
(3.126)
2
g (mN + M ) 2
H2 ,
, :
2 (mN + 4M )mN
H2 = 0
.
(3.127)
2 g (mN + 2 M ) 2
,
k :
H
(mN + 4M )(mN + M ) 2
k= 2 =
.
(3.128)
H
(mN + 2M )3
, mN = M, :
20
k=
.
(3.129)
27
3.4.
1
m (
) . u m1. ?
, .
m1
m1
: 1 = +
u , 2 = , 3 =
u.
m + m1
m + m1

3.

113

2
m0, k l0 .

m .
.
m0
.
: lmax = l0 + l, lmin = l0 l, l = m
(m0 + m)(2m0 + m)k
3
l,
,
m 2m . ( 2m) .
,
?
3m
: x =
l.
2(3m + M )
4
, M, , m,
U . .
m
M
U , =
U.
: =
m+M
m+M
5
.
: ?
: , .

114

6
m1, m2 m3
. , m1 c
m2. , ?
: m2 = m1m3 .
7
M a. m b < a
, .
b

m

M
a
?
.
m
( a b) .
: x =
m+M
8
m ,
/2, , = 30 .
, M/m = 5?
E k
1 m
2
=1
+ sin 2 = .
:
E0k
cos 2 M
5
9
m1
m2. .

3.

115

E k
4m1m2
=
.
k
E0
(m1 + m2 )2

10
m1 m2.
,
.
E k
2m1
:
=
.
k
m1 + m2
E0
11
m1
m2
, = 60.
.
m
: 1 = 2 .
m2
12
-
E 0 = 1 ,
= 60 .
,
E He - E .

: EHe = E0 sin 2 = 0,75 , E = E0 cos 2 = 0,25 .

116

4
.
4.1.
S S', . S' S - R(t ) S'
(t ) S' (. 4.1).

(t )

S'

R(t )

r (t ) M
r (t )

O'

O
. 4.1.


S S'.

d
d

dc
c

O'
. 4.2. c ,
S'.

S' (
) d (. 4.2).
,
d d .
c , S'. . 4.2
c :
dc = d c sin ,
(4.1)

dc = [d, c ]
(4.2)

4.

117

d
c& = c [c ] ,
(4.3)
dt
d

.
dt
- r (t )
M S - R(t )
S' S - r (t )
M S (. 4.1):
(4.4)
r (t ) = R(t ) + r (t ) .
(4.4)
S.
(. 1),
, :
( x i + y j + z k )
= r& S = R& + r& S = V +
=
S
t
S

i
j
k
=
= V + x& S' i + y& S' j + z& S' k + x
+ y
+ z
t S
t S
t S

= V + + (x[i ] + y[j ] + z[k ]) = V + + [r ] ,


[r ]
a = & S = V& + & S +
=
S
t S
= A + a + [] + [& r ] + [, + [r ]] =

(4.5)

= A + a + 2[] + [& r ] + [[r ]] .


(4.6)
S S'
S S , V A
S S.
( )
- r (t ) r (t ) , (t ) (t ) , a (t ) a (t ) S S':
r = R + r ,
(4.7)
[43
=V
+2
r ] +
{
= + ,
(4.8)
1
4

118

a = A + [& r ] +

]]
[1

4
r3
4[2

14444
244
443

[2
+ 21
3
] +

a{

= a + a + a .

(4.9)

= V + [r ]

; a = 2[] -

a = [[r ]]

,
a = A + [& r ] + [[r ]] a .
S',

r = R + r ,
(4.10)
[4
= = V
+2
r ] ,
(4.11)
1
4
3

a = a = A + [& r ] +

]]
[1

4
r3
4[2

(4.12)

14444244443



S (. 2). M, F i , S 2-
:
ma = Fi .
(4.13)
i

(4.13) (4.9)

S' :
mA + m[& r ] + m[[r ]] + 2m[] + ma = Fi ,
i

ma = Fi mA m[& r ] m[[r ]] 2m[] ,


14243 14243
i

1444424
4443

(4.14)

4.

ma = Fi + F + F .

119

(4.15)


S'. ,
, "" , , :

F = mA m[& r ] m[[r ]] = mA m[& r ] + F ,
(4.16)
14243


F = 2m[] .

(4.17)

, , ,
.
"" , , ,
(. 2).
S',
.
S'.
, , , (. 3). , ,
""
.
4.2.
4.2.1.


. :

120

1) ,
2) .
4.2.2.


I. .
1. , .
2.
( ).
3. ,
, .
4. (
).
II. .
1.
.
2.
, . 3.
3. ,
.
4. .
5.
.
III. .
1. .
2. ( , ,
).
3. .

4.

121

.

. I.3 I.5 ,
. II.2 .
. I.3 II.2 ( ), .
II.1 II.4 .
4.3.
4.1
( )
m,
l0, . F, (. . 4.3).

F
X, X'

. 4.3

lmax. k.

I. F
(. . 4.3),
. : , , ,
,
+ .
X X'

122

F (. 4.3). F .
F. .
II. (. (3.6)
3), + :
F
a =
.
(4.18)
2m
(4.16),
, :
F = ma .
(4.19)

+

(. (3.39) 3):
k (lmax l0 ) 2
(4.20)
= F x1 + F F x2 .
2
F , x1 x2 (
) .
(4.20)
.
, , , ,

. (4.20) , ( ).
III. (4.18) (4.20)
x2 x1 = lmax l0 ,
:
F
k=
.
(4.21)
lmax l0
,
3 ( 3.5).

4.

123

,
(3.5) (4.1), .
,
.
(8.11) 8.
4.2
( )
l m
, a g
(. . 4.4).

O
O'
y0

X'

Y'

mg

. 4.4

.
. .
1
I. X'O'Y',
(. 4.4), :
mg , T

124

F = ma , .
.
II. m
:
ma = ma sin mg sin ,
(4.22)

(. 4.4).

(. (1.19) 1):
a = & = &&l ,
(4.23)
.
III. (4.22) (4.23) :
g a
&& +
sin = 0 .
(4.24)
l
:
,
a=g.
sin
(4.24) :
&& + 2 = 0 ,
(4.25)
= ( g a ) / l , , .
,
(4.25)
(t ) = 0 cos(t + 0 ) ,
(4.26)
0 0
.
,
, (4.24)
(4.27)
&& = 0 .

4.

125

,
, , & , . :
(t ) = 0 + & t ,
(4.28)
0 .

(4.24),
(. (4.26) (4.28)).
2
XOY (. . 4.4) x, y X'O'Y' :
x = l sin ,
(4.29)
y = y0 + l cos ,
y0 O' X'O'Y'
XOY .

, (4.29):
&x& = l& 2 sin + l&& cos ,
(4.30)
&y& = &y&0 l& 2 cos l&& sin .
X Y
(. (2.2) 2):
m&x& = T sin ,
(4.31)
m&y& = mg T cos .
, &y&0 = a , (4.30) (4.31) :
g a
&& +
sin = 0 .
(4.32)
l

126

, (4.32) (4.24),

(4.26) (4.28) .
,
.
4.3
( )
R, (.
. 4.5). , . 0
H , .

I. , .
a . , mg , N
F = ma , . 4.5.

Y'
N

mg

n
. 4.5

X'

4.

127

. ,
.
II.

:

= mg cos N ma sin ,
(4.33)
R
m& = mg sin + ma cos ,
(4.34)
,
,
(. . 4.5).
,
, :
N =0.
(4.35)
(4.33) (4.35) :
(t = 0) = 0 , (t = 0) = 0 .
(4.36)
(4.33) (4.36), , .
,
. ,
.
( ) :
m

0
m02
mgR(1 cos 0 ) = ma cos R d ,
2
0

(4.37)

0 , 0 .
III. (4.33), (4.35) (4.37) :
02 = R( g cos 0 a sin 0 ) ,
(4.38)

128

02 = 2 R( g (1 cos 0 ) + a sin 0 ) .

(4.39)

(4.39)

02

Rg = Rg cos 0 + Ra sin 0 .
(4.40)
2
(4.38) (4.40)
:
2
0 =
gR .
(4.41)
3
0, ,
(4.38) (4.41) :
2 g 2 + a 5 g 2 + 9a 2
.
(4.42)
3( g 2 + a 2 )
H,
, :
cos 0 =

2 g 2 + a 5 g 2 + 9a 2
R.
(4.43)
3( g 2 + a 2 )
, a = 0 (4.43) 2
H = R .
3
H = R cos 0 =

4.4
( )
,
(. . 4.6).
Z'

F
X'

mg
. 4.6

Y'

4.

129


V . F, , r .

I. X'Y'Z',
,
Z'
, Y' V (. 4.6). mg , F ( ),
,
F (. (4.16))

F = m[[r ]] ,
(. (4.17))
F = 2m[V ] .

(4.44)
(4.45)

.
II.

.
:
0 = mg + F + F + F .
(4.46)
(4.46)
X'Y'Z', . 4.6, :
0 = Fx ' + F ,
0 = Fy ' + F ,

(4.47)

0 = Fz ' mg .
, .
III. (4.47)
(4.44) (4.45),
X'Y'Z'
:

130

Fx ' = 2mV ,
Fy ' = m 2 r ,

(4.48)

Fz ' = mg.
F
F = (mg ) 2 + ( m 2 r ) 2 + 4(mV ) 2 ,
(4.49)
F
X'Y'Z', , :
2mV
cos ' =
,
(mg ) 2 + (m 2 r ) 2 + 4(mV ) 2

cos ' =
cos ' =

m 2 r
(mg ) 2 + (m 2 r ) 2 + 4(mV ) 2
mg
(mg ) 2 + (m 2 r ) 2 + 4(mV ) 2

(4.50)

4.5
( )

Z'
V
, F
F
, .
F
,

,
, .
Y'
- X'

= 60 (. . 4.7),
V = 900 /, . 4.7
s = 1 .

I. X'Y'Z',
, Z' ,

4.

131

,
V (. 4.7). , ,
X'Y'Z' 2
=
, T = 24 .


F
F F ,
. 4.7 (4.16) (4.17). .
, , ,
. .
,
. . , , .
II. t
:
s
t= .
(4.51)
V
, ,
(. . 4.7)
a = 2 V sin .
(4.52)

at 2
l=
.
(4.53)
2
III. (4.51) (4.53),
:

132

l=

s 2 sin

7 .
V

V

.
F
(. . 4.8):

ma = F
.
(4.55)
(4.16) (4.55),
:
a = 2 r sin =

(4.54)
Z'

Y'

. 4.8

= 2 R cos sin .
(4.56)
(4.51) :
s
V a t = 2 R cos sin .
(4.57)
V
(4.57) V 0,01/ << V = 900 / ,
.
4.6
( )

. ,
h .
= 60 o .
L = 1 , V = 1/ ,
T = 24 .

g = 9,8 / 2 .

4.

133

I.
, . Y
( ), X ,
( ).
. 4.9 , mg ,
N
F .

Y
N

mg

. 4.9

, . . .
II. ,
, , ,
, :
mg + N + F = 0 .
(4.58)
(4.58) :
N sin + F = 0,
(4.59)
N cos mg = 0.
,
. 4.9,
h = L tg .
(4.60)

134

(. (4.17))
:
F = 2mV sin .
(4.61)
III. (4.59) (4.61), ,

2VL sin
h=
.
(4.62)
g

(4.62)
h 1.3 .
4.7
( )
L m, , , (. . 4.10).
.

I. , .
.
: mg, . .

- O
,

. ,
dF
dF ,
dl
.
mg
II. . 4.10
. -

4.

135

, , , , (. 4.10) , :
M mg + M = 0 .
(4.63)
(4.63) , .
:
L
(4.64)
M mg = mg sin .
2
dl, l
. (. (4.16)),
,
dF = Sdl 2 r ,
(4.65)
, S ,
r .

:
dM = dFl cos = Sdl 2l sin l cos .
(4.66)

L

M = dM = S 2 sin cos l 2 dl =
0

1
= L2 2 m sin cos .
(4.67)
3
III. (4.63), (4.64)
(4.67) , :
3g
cos =
,
(4.68)
2 L 2
sin = 0 .
(4.69)
3g

, >
2L

136

3g
(4.70)
2
2 L
, = 0,
(4.69),
3g
.
2L
( = 0).

= arccos

4.8
( )
L ,
(. . 4.11).
Y'

r'

X'
. 4.11

, t0 , l , , 0
. .

I. , . X' Y'
X'Y'Z' (. . 4.11). , . : mg,
, ,
. -

4.

137

. ,
,
.
II. , (. . 4.11):
ma' = m 2 r ' .
(4.71)

(4.71) a'
r' :
d ' d ' dr ' d '
a' =
=

=
' .
(4.72)
dt
dr ' dt dr '
III. (4.72), (4.71):
d '
'
= 2r ' .
(4.73)
dr '
(4.73)
:
'

r'

' d ' =
0

r ' dr ' .

(4.74)

(4.74), ' r':

' = r '2 l 2 .

(4.75)

dr '
= r '2 l 2 .
(4.76)
dt
(4.76) :
L

dr '

t0

= dt .

(4.77)
2
2
r

l
'
0
l
(4.77) :

L2
1 L
t0 = ln + 2 1 .
(4.78)

l
l

138

(4.75):
' = r '2 l 2 eX ' ,
(4.79)
eX ' X'.

(4.8) :

= [r ] + = r ' e Z ' + r '2 l 2 eX ' ,


(4.80)
eZ' Z'. ,
:

= 2r '2 l 2 .

(4.81)

4.91
( )
m M ,
. .

I. , , X' Y'
(. . 4.12
4.13).
R
Y'
Y'
N
ma 0
F
mg

Mg
. 4.12

Ma0

X'

X'

N
. 4.13

: mg,
F ( )
1

4.9 4.10, 2 6.

4.

139

N ,
ma0 (. 4.12), a0 , X'.
.
: Mg,
N F , R Ma0 (. 4.13).
II.
:
ma x ' = N sin F cos + ma0 ,
(4.82)
ma y ' = N cos + F sin mg ,

(4.83)

ax ' , a y ' a , :
a x ' = a cos , a y ' = a sin .

(4.84)

(. (6.30) 6)
, (. . 4.12), :
mr 2
&& = F r ,
(4.85)
2
&& , r .
(4.85) J mr 2
,
, J =
2
,
.
(
a) :
(4.86)
a = &&r .

':
0 = N sin + F cos + Ma0 .
(4.87)

140

III. (4.82) (4.87), :


mg sin 2
a0 =
.
(4.88)
3M + m(1 + 2 sin 2 )
, :
mg sin 2
a0 =
eX ,
(4.89)
3M + m(1 + 2 sin 2 )
eX
, X' .
4.10
( )
m R L
. a
(. 4.14), . V , ?

N
a

ma

X'

mg
. 4.14

I. , . X'
. : mg , N ,
F ( ) -

ma . .

4.

141

II.
':
m&x&' = ma F .
(4.90)
(. (6.30) 6)
, (. . 4.14), :
J&& = F R ,
(4.91)
&& , R .

mR 2
J=
(4.92)
2
(
&x& ) :
&x& = &&R .
(4.93)
:
(4.94)
V = &x&t .
&x&t 2
,
(4.95)
2
, .
III. (4.90) (4.93), :
2
&x&' = a .
(4.96)
3
,
, (4.94)
(4.96):
4
V =
aL .
(4.97)
3

L=

142

4.4.
1
F ,
m ? F , .


.
tg
: F ( M + m) g
> tg , tg 1 + tg
F.
2
F ,
m ? F , .


.
:
tg
tg +
< ctg ;
( M + m) g
F ( M + m) g
1 + tg
1 tg

4.

( M + m) g

143

tg
F ctg .
1 + tg

3
.
0. .
2
: a = ( g a0 ) sin .
3
4
R ,
.
m. ,
, F , ,
, . F
r .
2

2R
: F = m 2 r
1 .
r
5

,
(.

.).

,
k
.

l.


?

144

: =

g 2
mg
, , kl >
.
l
2

6
( ). = 60,
V0 = 100 /.
.
:
4 V 3 cos
x= 0 2
0,5 .
3
g
7
m V .
.
: F = 2mV sin ( ).
8
.
, .
.
? , m,
l. , .
2ml0
: V =
, 0 .
M +m
9

, , ? V0 = 100 /c, = 60.
:
2V cos
= 0
51 .
3g

4.

145

10
H
.
, .
.
2
2H
: x H
.
3
g
11
m . N
. :
, , ,
. R .

(R ) 2
.
: N = m g

146

5

5.1.
5.1.1.
()
I. :

. , (,
, ).
II. : ( )

.

, , (x,y,z)
t, .
-
(x,y,z,t) ( r ,t).

,
S,
, , .
.
A B,
S ( ):
t A + t 2A
tB = 1
.
(5.1)
2

5.

147

(. . 5.1) t1A A ( ) A, t B B B, t 2A
A B A.
S
tB

t2A
t1A

. 5.1. ,
S

5.1.2.

-
(. . 5.2).

Y'

S'

( r ,t)
( r ,t')
V
X

X'

Z'

. 5.2.
-

S' S
V X (. 5.2). , - ( r = 0 , t = 0) S ( r '= 0 , t' = 0)
S'. -

148

, -
(x,y,z,t) (x',y',z',t') :
x Vt
x + Vt

,
x=
,
x =

2
2
(
)
(
)
1

/
V
c
V
c
1

y = y,
y = y,

z = z,
(5.2)
z = z,

V
V

t 2 x
t + 2 x
t =
t =
c
c
;
;

2
2
(
)
(
)
1
/
V
c
V
c

1
/

x = (x Vt ),
x = (x + Vt ),
y = y,
y = y,

z = z,
(5.3)
z = z,

t = t V x ;
t = t + V x .
2

c2
c

1
1
V

=
> 1 - < 1 .

2
2
c

1
1 (V / c )
(5.3) x = x2 x1 , y = y2 y1 , z = z2 z1
x = x2 x1 , y = y2 y1 , z = z2 z1 , t = t 2 t1 t = t2 t1 :
x = (x Vt ),
x = (x + Vt ),
y = y,
y = y,

(5.4)
z
z

z = z ,

t = t V x ;
t = t + V x .
2

c
c2

, .

5.

149

5.1.3.
1.
(
) (
) .
2. " "
, , .
(5.4).
3. " "
,
S' ( "" S') (. 5.3).

Y'

S'

x1,t1

x2,t2

x1 , t1

x 2 , t 2

X
Z

X'

Z'

. 5.3.
-

,
S': x1 = x2 ,
x = x1 x2 = 0 . S
. (. (5.4)):

150

t =

t
1 (V / c )

= t t =

(5.5)

t' ,
S',
S', t S.
, ,
, ( ).
T0 t' ( S') S', T t S, ,
:
T
T0
= T0 , T0 = .
(5.6)
T=
2

1 (V / c )
4. " "
, S.
, , x2 x1 S' ,
X X' (. . 5.4).

Y'

S'

x1,t1

x2,t2

x1 , t1

x 2 , t 2

X'

Z'

. 5.4.
-

5.

151

, S: t1 = t2, t = t2 t1 = 0. S'
.
(5.4) x = x2 x1 x = x2 x1
:
x
x
x =
= x x =
.
(5.7)
2

1 (V / c )
, ,
, (
) .
(.
. 5.4), :
l
l
(5.8)
l0 =
= l l = 0 .
2

1 (V / c )
l0 x ( S',
); l x S,
V (
).
5.1.4. -
- :

S12

(x2 x1 )2 + ( y2 y1 )2 + (z2 z1 )2 c 2 (t2 t1 )2

= x 2 + y 2 + z 2 c 2 t 2 = r122 c 2t122 .

=
(5.9)

r12 = x 2 + y 2 + z 2 0
( ,

152

) t12 = t 0
.
(5.2),
, - ,
:
.
S12 = S12
(5.11)
, r12
t12 , .
- ,
S122 > 0 . : r12 > ct12.


1. , , . r12
:

= r12 ,
S12 = r122 c 2t122 = S12

(5.12)

r12 = r122 c 2t122 .


(5.13)
2. , .
3. , , .
4. , , :
r
S122 > 0 , r12 > ct 12 , = 12 > c .
(5.14)
t12
5.
( , , -

5.

153

,
),
, .
, S122 < 0 .
: r12 < ct12.


1. , ,

. t12
:

= ict12
,
S12 = r122 c 2t122 = S12

(5.15)
)2 ,
S122 = r122 c 2t122 = c 2 (t12
i
=
t12
r122 c 2t122 .
(5.16)
c
2. , =0.
: t12
3. , ,
.
4. , :
r
S122 < 0 , r12 < ct 12 , = 12 < c .
(5.17)
t12
5. ,
,
, ct12 > r12 S122 < 0 .
S122 = 0 .

154



1. ,
( )
:

S12 = r122 c 2t122 = 0 , r12 = 0 t 12 = 0 .


(5.18)
2. ,
, , .
3. , ,
.
. x = ct , , (5.4), :
V
V
V x

(5.19)
t = t 2 x = t 1 2 = t 1 m .
c
c

c t

, t
t
S' S.
4. , ,
:
r
S122 = 0 , r12 = ct 12 , = 12 = c .
(5.20)
t12
5. , ,
.
,
,
.
5.1.5. ()
(5.2) (. . 1.1 1)

5.

155

dx
d x

x = d t , x = d t ,

dy
d y

(5.21)
, y =
,
y =
dt
d t

dz
d z

z = d t ; z = d t ;

x V
+ V
x = x
,
,
x =

V
V

1 2 x
1 + 2 x
c
c

2
2
1 (V / c ) y
1 (V / c ) y

(5.22)
, y =
,
y =
V
V

1 2 x
1 + 2 x
c
c

2
2
= 1 (V / c ) z ; = 1 (V / c ) z .
z
z
V
V
1
1 + 2 x

2 x
c
c


, . .

.
.
5.2.
5.2.1.


. :
1) (" ", " " "
");
2) - ;

156

3) .
, ,
.
5.2.2.

I. .
1. , ( ).
2.
( ).
3. .
4. - .
II. .
1. (
(1)).
2. -
( (2)).
3. (
(3)).
4. (, ).
III. .
1. .
2. ( ).
3. .

5.

157

5.3.
5.1
( )
,
S.
t = 20 . S', ,
t' = 25 . .

I. , S', V X S. ,
, .

. 5.4
(.
. 5.1. ). ,
.
- , ( x1 , t1 )
( x2 , t 2 ) S' ( x1 , t1 ) ( x2 , t 2 ) S. , x1 = x2 .
II. l0 ,
l0 = x2 x1 .
V,
l0 = Vt ,
(5.23)
t = t 2 t1
S'.
V
t t = t 2 t1 (. (5.5)):
t
t =
.
(5.24)
2
1 (V / c )
,
t , S ( " ").
III. (5.24), V:

158
2

t
V = c 1
(5.25)
.
t
(5.23) (5.25)
(
S')
2

t
l0 = Vt = ct 1 .
(5.26)
t '
(5.26) t = 20 t' = 25 ,
, c 3 108 / ,
:
l0 = 4,5 .
5.2
( )

t0 = 10 . , , ,
t = 20 ?

I. . S'
. , S' S
V. . 5.3 (.
. 5.1. ). , .
-
S ( x1 , t1 ) ( x 2 , t2 ), S'
( x1 , t1 ) ( x 2 , t2 ), x1 = x2 . t0 S',
:
t0 = t = t 2 t1
(5.27)
II. l, S
:
l = Vt .
(5.28)

5.

159

(, , S')
" ".
S' , , (5.5), S' .
, :
t
2
(5.29)
t0 =
= t 1 (V / c ) .

III. (5.29), S':


2

t
V = c 1 0 .
(5.30)
t
(5.30)
(5.28), l,
S:
2

t
l = ct 1 0 .
(5.31)
t
(5.31) t0 t ,
, :
l = 5,2 .
5.3
( )
S' S
X V = 0,9c.
200 , 100 , 0 , 100 200 . , ,
S x = 0 ,
, S'
x' = 0 . , , " " , S, S'.

160

I. Aj (j =1, 2, 3, 4, 5) ,
t A j = 0 S -

j- , S'
x A j ( 200 , 100 , 0 , 100 200
) S'. Bk (k =1, 2, 3, 4, 5)
k- S, x Bk
(200 , 100 , 0 , 100 200 ) S
t B k = 0 S'. , Aj S.
, Bj
S'.
II. - Aj Bk
S S' (. (5.3)),
tA j = 0 tB k = 0 :

xA j = xA j , tA j =

V
xA j ;
c2

(5.32)

V
xB k .
(5.33)
c2
S', xA j

xB k = xB k , tB k =

tA j

S' S, t B k S, xB k S S'.
(5.32), (5.33)
-:
1
=
.
(5.34)
2
1 (V / c )
III. (5.32) (5.34) -
( tA j , xA j tB k , xB k ) A B
V S' S, :

5.

xA j =

161

xA j = xA j 1 (V / c ) , tA j =
2

V
xA ;
c2 j

(5.35)

V
xB .
(5.36)

c2 k

, xA j xB k
xB k =

xB k = xB k 1 (V / c ) , tB k =
2

.
5.5 , S,
. 5.6 S'.
V

S
S
200

100

+100

+200 X

+100

+200 X

. 5.5

S
200

100

S
V
. 5.6

,
" " ,
, (. (5.35)
(5.36)). . -

162


, ,
(. . 5.5 5.6). ,
, , , (.
. 5.5 5.6).
,
,
. V = 0,9c,
xA 1 = xB1 = 200 xA 5 = xB5 = 200 , :

V
V
V
x + 2 xA 1 = 2 xA 1 xA 5 =
2 A5
c
c
c
= 1,2 10 6 c = 1,2

tA 5 t A 1 =

V
V
V
x 2 xB1 = 2 xB 5 xB1 =
2 B5
c
c
c
= 1,2 10 6 c = 1,2 .

tB 5 tB1 =

(5.37)

(5.38)

5.4
( )
, L = 3 ,
V = 5106 /. ,
. t
?
,
.

I. , S,
, .
S'
. , S', V X S. , , :

5.

163

V
;
;
.
II.
S
L
TAB = TBC = .
(5.39)
V
, ,
, , . , , , , , ,
" ". , (5.6),
S':
L
= TBC
=
.
(5.40)
TAB
V
t, , :
+ TBC
)
t = (TAB + TBC ) (TAB
(5.41)
III. (5.39) (5.41) t
:
2L
t =
1 1 2 .
(5.42)
V
V
0.33 10 2 << 1 ,

c
2
1
V
1 1 2 2 = 2 , , , 2
2c

2 L V 2 LV
t

=
0,05 18 .
(5.43)
V 2c 2 c 2

5.5
( )
, V = c/2
S, l0 = 1 . S',
, -

164

0 = 45 (. 5.7).
l S.

Y'

S'
V
l0

0
X

X'

. 5.7

I. ,
S, . , S.
-
S ( x1 , y1 , t1 ) ( x 2 , y2 , t2 ), t1 = t2 .
S' ,
-
( x1 , y1 , t1 ) ( x 2 , y2 , t2 ).
II. ( S') :
2
2
2
l0 = (x) + (y) .
(5.44)
x y
l0 0 S' :
x = l0 cos 0 y = l0 sin 0 .
(5.45)
A B S, S
" " (. . 5.1.3). (5.7):
1
(5.46)
x = x .

5.

165

(5.4)
y y1 y2 : y = y .
l S:
2
2
l 2 = (x ) + (y ) .
(5.47)
S x
y :
y
(5.48)
.
x
III. (5.47) (5.45) (5.46),
S:

= arctg

cos 2 0

V
= l0 1 cos 2 0 .
(5.49)
2

c
(5.46) (5.48)
:

1
y
= arctg
tg 0 .
(5.50)
= arctg
1 (V / c )2

(5.49) (5.50) l0 V, :
l 0,935 , 49 .
(5.51)
l = l0 sin 2 0 +

5.6
( )
V = 0,6
. ,
,
. ,
.
= 60 .

I. S' , S (. 5.8).

166

Y'
A
L/2

S'
V

B
L/2

X, X'

. 5.8

, . S t1
x1 , . , ,
t2 x2.
II. , S, :
L
ct1 = Vt1 ,
(5.52)
2
L
ct2 = + Vt 2 .
(5.53)
2

S :
L 1
1
LV
.
(5.54)
t = t 2 t1 =

= 2
2 c V c +V c V 2
L ,
(5.55)
L = c .
" ". S'
, , (5.6), :

t =

V
= t 1 .

(5.56)

5.

167

III. (5.54) (5.56), :


2

LV
c2 V 2
cV
V
t = t 1 = 2

=
=
2
c
c V
c
c c2 V 2
V
=
.
(5.57)
c2 V 2
(5.57) V = 0,6 ,
= 60 ., :
t ' = 45 .
5.7
( )
, ,
1 = 1 .
T = 1,5 .
2 = 15 .
1, 2 T .
T, .

I. :
;
;
1 ;
1 ;
2 ;
2 ;
.
. 5.9 ( ) , ( t)
( t').

168

LA/V
LA/c

T
tB

A B

A2 B2

A1 B1

t'

2
T = tA
tB/

A B

A2 B2

A1 B1

2
T
. 5.9

, , , , , .
,
, 1, 2 T.
" " (. , (5.6)) , , , (. . 5.9).
II.
2 , ,
tA , , :
L + (LA Vt A )
,
(5.58)
tA = A
c
LA ,
, .

5.

169

, ,
" ", :
tA = T .
(5.59)
, (5.58), tB B B2 ,
:
L + (LB VtB )
,
(5.60)
tB = B
c
LB ,
B , .
. 5.9, t B
1, 2 T:
tB = T 1 + 2 .
(5.61)

( A
B) , ( 1 ):
LA LB = V 1 .
(5.62)
A1 C
L
L
T = A A .
(5.63)
V
c
III. (5.58) (5.63), V T,

:

V =c 1 2 ,
(5.64)
1 + 2
T = T

1 2
.
1 2

(5.65)

, , :
V = 0,6c , T 1 .

170

5.8
( )
(. . 5.10).
, 1 = 1 , . 2 = 0,5 = 1 . l0
U .

S'

l0
. 5.10

I. S ,
, S', S U (. 5.10).
:
, ;
1 1 ;
C , S
( );
1 , .
. 5.11 , ( t)
( t').
, ,
, ,
, .

5.

171

tB
tA
A
C

2
A1 1

B1

B1

t'


tC

CA

A1 1

. 5.11

. 5.11
, , 1, 2 .
, , A B S ( 1), A1 B1 S'
( 2), A1 C1 S' ( ).
" " (.
, (5.6)) ,
,
, (. . 5.11),
1
=
.
2
1 (U / c )
II.
( A)
LA S. tA A A1
:
L
tA = A .
(5.66)
c +U
( ),
( A)
tC :

172

LA + l0
.
(5.67)
c +U
tB B B1 S
1 :
L U 1
tB = A
.
(5.68)
c +U
. 5.11,
:
tC = t A + ,
(5.69)
tB = tA 1 + 2 .
(5.70)
III. (5.66) (5.70)
l0 U:
tC =

1
,
2

(5.71)

12 2 2
c.
2 2 + 12

(5.72)

l0 = c

U=

(5.71) (5.72) 1,
2 , l0 U:
3
(5.73)
l0 = 600 , U = c .
5
5.9
( - )
- : x1 = 0;
t1 = 0 ( ) x2 = 5 ; t2 = 108 c ( ). :
1) x min ,

V;
2) t min ,
V;

5.

3)
.

173

I. - S.
- (5.9)
:
2
S12 = x 2 c 2 t 2 = 16 2 > 0 ,
(5.74)
x = x2 x1 t = t 2 t1 .
2

S12 > 0 , -,
- (.
. 5.1.4. - )
II. - ), x S',
( S12 = S12
t = 0 :
= S12 =
x min = S12

(x2 x1 )2 c 2 (t2 t1 )2 .

(5.75)

, t min = 0 S'.
S' (5.4):
x

(5.76)
t = t
.
c

III. (5.76) t = 0 , :
t t
t t
V ct
= =
= c 2 1 V = c2 2 1 .
(5.77)
x2 x1
x2 x1
c x
-
(5.75) (5.77),
:
3
3
x min = 4 , = V = c .
5
5

174

5.10
( )
l0

V S (. 5.12).
, .

Y' S'
1

X'
. 5.12

I. S' (.
. 5.12). S 1
V. S
2 = V .
S', . S'
. S' .
II.
" " S',
, :
2


= l0 1 2 ,
(5.78)

c
2
S'. ,
(5.21):
l2 =

l0

5.

2 V

175

2V
.
(5.79)
2
V
V
1 2 2
1+ 2
c
c
III. 2 (5.79)
(5.78), :

2 =

l2 = l0 1

4V 2

= l0

c2 V 2
.
c2 + V 2

(5.80)
2
V2 2
1 + 2 c

S' ,
,
:
c2 V 2
l1 = l0 2
.
(5.81)
c +V 2
5.11
( )

Y
(. . 5.13). U , Y,
= 90 ?
Y

U
U

. 5.13

I. Y S ,
(. 5.13), X .
S , -

176

U S,
X' Y' X Y.
II. S',
:
1x = 2 x = 0 , 1y = c 2 y = c .
(5.82)
()
(5.22) S:
U + 1x
U + 2 x
,
(5.83)
, 2 x =
1x =
U
U
1 + 21x
1 + 22 x
c
c
2

U
U
1 1y
1 2 y
c
c
1 y =
, 2 y =
.
(5.84)
U
U
1 + 21x
1 + 22 x
c
c

= 90 ,
:
1x = 1 y 2 x = 2 y .
(5.85)
III.
S (5.83) (5.84) (5.82):
2

U
U
1x = 2 = U , 1 y = c 1 , 2 y = c 1 .
(5.86)
c
c

(5.85), , U , , ,
90 :
2

U
U = c 1 .
c

c
U=
.
2

(5.87)

(5.88)

5.

177

5.12
( )
X' S' U = 0,7c , Y' (. . 5.14). S'
V = 0,6c S ,
X'.
S.

Y'

S'
V
A

B
U'
X

X'

. 5.14

I. C
D, ,
X' S'. C D S
( x1 , y1 , t1 ) ( x2 , y 2 t2 ), S' ( x1 , y1 , t1 )

( x2 , y 2 , t2 ) (. . 5.15).
Y

Y'

S'
V

x1

x 2
U'

. 5.15

X'

178

II. C D S' ,
t = t2 t1 = 0 .
(5.89)
S' S C D .
t C D S
(5.89) :
V
t + 2 x
c
t = t 2 t1 =
.
(5.90)
2
1 (V / c )
x = x2 x1 > 0 , t = t2 t1 > 0 . ,
S A B
y = y0 ,
, t B (. . 5.16).
Y

y0

S
B
A

Uy

x1

x2

. 5.16

, S
. ,
y = y0,

y = U y t ,
(5.91)
U y , Y
S. (5.7), S
x = x2 x1 :
x
2
x =
= x 1 (V / c ) .
(5.92)

5.

179

S
:
y
(5.93)
= arctg .
x
U y Y S
() (5.22)

1 (V / c ) U y
Uy =
.
V
1 + 2 U x
c

U x = 0 , U y = U .
2

(5.94)

(5.95)

III. S
, (5.90) (5.95):

U V
.
= arctg
(5.96)
c 2 1 (V / c )2

(5.96)
S', , :
27,7 .

,
90
60
30

= 36,9
27,7

0
0

0.2

0.4

0.6
V/c

. 5.17

0.8

180

(5.96) V S'
U'. . 5.17 (V / c )
U / c . 1 U / c = 0,999 , 2 U / c = 0,7 , 3 U / c = 0,3 .
, S' S
X ,
1 . 5.17.
5.4.
1
, V = c/2, l = 1
= 45.

)(

: l0 = l 1 (V / c ) sin 2 1 (V / c ) = 1,08 .
2

2
l0 . , ,

t.
?
2l0c 2 t
.
: V =
(ct )2 + l02
3
, S L, , X . S' S V = c/2 , Y. ' X' S' L'
.

5.

: tg =

181

3
3 + cos 2
tg , L = L
.
2
2

4
0 ( )
.
, , ,
,
( ). ?

: V = Tc

4 02 + T 2 .

5
S'
S V = c/2 , X. S
U = c/2 X. U
S'. ,
X' S'.
c
3 sin
: U =
8 + 8 cos sin 2 , tg =
.
4 + cos
2(1 + cos )
6

V1 > V2 .

1 .
2
?
c + V1 c V2

.
: 2 = 1
c V1 c + V2
7
l = 6105
= 1

182

. V , ,
, ?
c 2
= 1,5 108 / .
: V =
l
8
S, c / 2
X S,
c / 2 X'. ,
X S.
3 sin
: tg =
2 cos + 2
9
V

, . l
, .
V
.
: l =
2
1 (V / c )
10
,
1 , 2 . , ,
1 2 . L , ?
( , .)
+ 2 2
T,
: L = 1 1
1 + 2
1
1
2 =
.
1 =
2
2
1 (1 / c )
1 ( 2 / c )

5.

183

11
.
V,
, , = c 3 .
2
= 0,6c .
: V =
2
1 + ( / c )

184

6


6.1.
6.1.1.
( ),
.
S. S' . ,

S'.
, S S'
(. 4):
r = R + r' ,
(6.1)
= V + [r' ] + ' ,
(6.2)
&
a = A + [r' ] + [[r' ]] + 2[' ] + a ' .
(6.3)
r (t ) , r ' (t ) -, (t ) , (t ) a(t ) ,
a(t )
S S' ; R(t ) , V (t ) A(t ) , S',

; (t ) & (t )
S' ( ) , S' (. 6.1).
M (. . 6.1) ( S'), , (t ) a(t ) , :
r = R + r' ,
(6.4)
= V + [r' ] ,
(6.5)
a = A + [& r' ] + [[r' ]] .
(6.6)

6.

185

S'
r

M
r'

O'

O
. 6.1. - M
S S',

(6.5)
( ) .

( )
( V )
(
).
, , , . .
(
S')
(
S' ) R(t )
( S')
(t ) .
, ,
.

186

. , ,
, ,
.

, .
.

,
,
, .
.
(t ) a (t ) P (. . 6.2).

S'
r

M
r'

O'
O
. 6.2.

, n const
,
(n) = n dr = 0 , (n) = (n& ) + (n& ) = (na ) = 0 .
(6.7)
t
dt

6.

187

&

.
,
(6.5) .
V (t ) r ' (t ) , :
(n) = ({
nV ) + (n[r ']) = 0 ,
123
=0

=0

n V , n || V .
(6.8)
(6.6)
. A(t ) r ' (t ) , :
(na ) = ({
nA) + (n[& r ']) + (n[[r ']]) = 0 ,
124
3 14243
=0

=0

= 0, n||

(6.9)
n A n || & .
, r0 ,
= V + [r '] = [r0 ] .
(6.10)
, (6.10), :
V + [, r 'r0 ] = 0 , d r 'r0 , [n,V + [d ]] = 0 .
d , d n , :
(nd ) d (n) = [nV ] d (n) = 0 ,
[nV ] + [n[d ]] = [nV ] +
123
d=

[nV ] = [nV ] .
(n)

=0

(6.11)

, , - R0 , (6.10):
r = R + r ' = R0 + r0 ,
[nV ] .
(6.12)
R0 = R + r ' r0 = R + d = R +

,
,
""
.

188

. 6.3 ,
, .

Ri
R2
R1

. 6.3.

. 6.4, :

=K=

,
(6.13)
R1 R2
Ri
i ,
Ri
.
,

, "" , ..

(6.10).
- R0 ,
, (. (6.12)).


,
.

6.

189

,
( S') , S, , ( R = 0 , V = 0 , A = 0 ), :
r = R + r' ,
(6.14)
= [r' ] ,
(6.15)
a = [& r' ] + [[r' ]] .
(6.16)
(6.15) , ,
.
,
,
, . , .
, .
S1 S
, , 1 S1, (6.5):
=V
(6.17)
{ + [r '] + ' = [r '] + [1r '] = [ + 1 , r '] = [r '] ,
=0

= + 1 .
(6.18)

S, .
, , ,
1
2 .
,
, , "" .

190

6.1.2.
L
r p :
L = [rp] .
(6.19)
L Li ,
:
L = Li .
(6.20)
i

L L0 O'
(. . 6.4).

mi
ri

S'

ri '

O'

O
. 6.4. -
S S'

- i- ri :
(6.21)
ri = r + ri ,
r - , ri
- .
(6.19) (6.20) :

6.

191

dr

(6.22)
L = ri , mi i .
dt
i
(6.22) (6.21):
dr
dr
dr

L = r + ri, mi i = r , mi i + ri, mi i =
dt i
dt i
dt
i
= [r , pi ] + [ri, pi ] = [r , p ] + L0 L + L0 ,
(6.23)
i

pi i- ,
p mi i = pi
i

L0 [ri, pi ] i

O' L [r , p ]
O,
.
S', O'
, (. . 6.4). (. . 1.1 (1.26) 1) :
(6.24)
i = + i ,
(6.24) i-
:
(6.25)
pi = mi + pi .
(6.25) (6.23), :
L = [r , p ] + [ri, pi ] = [r , p ] + [ri, mi ] + [ri, p] =
i

= [r , p ] + mi ri, + [ri, p] .
(6.26)
i
i
(. 3)
mi ri = 0 , , (6.26)
i

192


:
L = [r , p ] + [ri, p] = [r , p ] + L0 ,
(6.27)
i

L0 [ri, p] i

O' S'.
,

. ,
(. (6.23)
(6.27)).

, .

(6.15), ,
:
L = [ri , mi i ] = mi [ri , [ri ]] = mi ri2 ri (ri ) =

mi ri2 xi2
mi xi yi
i
i
= mi yi xi
m
i ri2 yi2

i
i

mi zi yi
m
z
x
i i i
i
i

J xx J xy J xz x

= J yx J yy J yz y = J .

J zx J zy J zz z

mi xi zi
x
i
mi yi zi y =

i
2
2 z
mi ri zi
i

(6.28)

6.

193

ri - i- mi , i

x

, = y

z
J xx J xy J xz

J = J yx J yy J yz

J zx J zy J zz
.
J xx , J yy , J zz , J xy = J yx , J xz = J zx ,

J yz = J zy .
.
, , :
Jx 0 0

J = 0 Jy 0 .
(6.29)
0 0 J
z

,
J x , J y J z
, :
L = J ,
(6.30)
L L ( = x, y, z ).
,
J x = J y = J z J (6.28) -

:
L = J = J .
(6.31)
S'
, :

194

L = L = J 0 .
(6.32)
J 0
.
,
:
L = L + J 0 .
(6.33)
Ln , :
Ln = (nL ) ,
(6.34)
n , .
Ln L0, n ,
, , . n
L = L + L0 , :

Ln = L, n + L0, n ,

(6.35)

L, n .
,
.
M - r F :
M = [rF ] .
(6.36)
M n
, :
M n = (nM ) .
(6.37)

6.

195


,
, . ,


, .

, ,
,
.
Fp {Fi } , -

, , Fp = Fi ;
i

,
.

, .
, ,
(. 4) ,
.

.

,
(, ). .
(. 3)
.

196

( )
L M ex , :
dL
= M ex ,
(6.38)
dt
M ex = M iex = ri Fi ex .

( )
Ln

M nex , :
dLn
= M nex .
(6.39)
dt
, ,
, :
J = mi ri2 .
(6.40)
i


, :
J = r 2 dm = r 2 dV ,
(6.41)
V

r dV dm ,
.
J
J 0 , :
J = J 0 + ma 2 .
(6.42)

6.

197

,
,
.
, ,
(6.41) :
l/2

J =

x2

l/2

ml 2
m
dx =
,
12
l

(6.43)

m , l , x .
() (6.41) :
L/2 R 2

J =

2
r rddrdz =

L/2 0 0

mR 2
,
2

(6.44)

m, R L , , r z .
, , (6.41) :
R 2

J =

(r sin ) r sinddrd = 5 mR
2

(6.45)

0 0 0

m R , r, .

n ,
,
:
- (. 3)
ma = F ex ;
(6.46)
- n ,
d
J 0, n
= M 0,exn .
(6.47)
dt

198

m , a , F ex
, , J 0, n
,
,
ex
, M 0,n

, , .




, :
d
Jn
= M nex ,
(6.48)
dt
J n , -

, M nex , .
, (6.45) , .
6.2.
6.2.1.


.
1. :
)
,
)
,
) ,

6.

199

)
( ) .
2. :
)
,
)
,
)
( , ).
6.2.2.

I. .
1. , .
2. , (), ( ).
3. .
4. ( )
.
5.
() .
II. .
1. .
2. .
3. ,
.
4. , .
5.
.

200

6. .
7.
.
III. .
1. .
2. ( , ,
).
3. .
.

. I.5 . II.1 II.5 .
6.3.
6.3.1.
6.1
R . , , A . B, ,
B = 5 A . ,
0
M (. 6.5).
A

y0

0 B

yM

B
X

. 6.5

6.

201

I. ,
. X Y
, . 6.5.
, "" ,
, (.
. 6.1. ).
, , , . ,
.
, ,
.
y (.
. 6.5) .
, 0 , , X Y.
II. (6.2), (. . 6.1. ).
0

A B R .

(. . 6.5):
A = 0 + R ,
(6.49)

B = 0 2 + 2 R 2 .

(6.50)
.
(. . 6.5):

202

R
,
(6.51)
0
R
tg =
,
(6.52)
R yM
B
.
III. (6.49) (6.50)
:

2 2
(6.53)
2 A + A 2 B = 0.
tg =

R
2R
, :
1

2
2
1, 2 = A
2 B A .
(6.54)
2R 2R
B = 5 A , :

1 = 4

2 = 3

.
(6.55)
R
R
(6.49)
:
01 = 3 A 02 = 4 A .
(6.56)
(6.51) (6.52)
:
yM = R

0
.

(6.57)

(6.55) (6.56) (6.57), :


7
7
yM1 = R yM2 = R .
(6.58)
4
3
, .
1. X, ,

A, (. . 6.6):
7

01 = 3 A , 1 = 4 A , yM1 = R .
4
R

6.

203

2. X,
, A (.
. 6.7):
7

02 = 4 A , 2 = 3 A , yM2 = R .
3
R

M
0

0
B

X
. 6.6

A
O

R B
0

B
0

X
. 6.7

6.2
, h = 4 r = 3 ,
,
O (. 6.8).
, ,
OZ T = 3 .

204

Y
h

r
A
. 6.8

I.
, . Z , O (. . 6.8).
( ,
OC)
1
OC 2 Z. , OA ,
, .
,
= 1 + 2 .
II. C
- Z
Z
R
C
R (. . 6.9).

f
- O
A
:
. 6.9
2
C = 2 R =
R.
(6.59)
T
,
C OA

6.

205

. , C :
C = f ,
(6.60)
. 6.9, R f :
r
h
,
(6.61)
=
f
h2 f 2
R = h2 f 2 .
(6.62)
III. (6.59) (6.60), :
R 2
=
.
(6.63)
f T
(6.61) (6.62) R f:
h2
rh
, R=
.
(6.64)
f =
r 2 + h2
r 2 + h2
:
h 2
(6.65)
=
= 2,79 /.
r T
. , ,
:
1 + 2 = .
(6.66)
1 . , A, ,
,
:
= OA 2 1r = 0 .
(6.67)
,
OZ (6.67) . 6.9

h2 + r 2
.
(6.68)
r
1 , 2 , . 6.10.

1 = 2

206

1
Z

. 6.10

1 OC, 2 Z
, . (. . 6.10).
(6.67) (6.68), ,
2
h2 + r 2
2 h

12 2 2 = 2 2
1
=

= 2 .
(6.69)
2
2
2

r
r

6.3
r1 r2 (r1 < r2)
1 2
( 1 > 2 ).
r3 = (r2 r1)/2, (. 6.11).
0
.

I. B A . ,
, A
B , (. . 6.11).

6.

r1

O
r2

r3

207

. 6.11

II.
1 2 , A B :
A = 2 r2 ,
(6.70)
B = 1r1 .
(6.71)

(.
. 6.1. ),
. , O, B A.
O B rx O, :
A = (r1 rx + 2r3 ) ,
(6.72)
B = (r1 rx ) ,
(6.73)
:
0 = (r1 rx + r3 ) .
(6.74)
III. (6.70) (6.73) , :
r r
= 2 2 11 .
(6.75)
2r3
(6.75) (6.74), :

208

0 =

1r1 + 2 r2
2

(6.76)

6.3.2.
6.4
m R. a .

I. , .
M (. . 6.12)

X
mg

. 6.12


( ).
, , X
, Y , XY ,
(. 6.12).
mg , F ( )
N (. 6.12).

6.

209

II. (
(6.48)) ,
(. . 6.12), :
d
J
= mgR sin ,
(6.77)
dt
, J .

.
d
( a
dt
):
d
.
(6.78)
a=R
dt
, - (6.42):
J = J 0 + mR 2 ,
(6.79)
J 0 , (6.45),
2
J 0 = mR 2 .
(6.80)
5
III. (6.77) (6.79) , :
1
(6.81)
a=
g sin .
J0
1+
mR 2
(6.80) ,
:
5
a = g sin .
(6.82)
7
, . , -

210

(. . 2.1.2 2) :
F N .
(6.83)
.
, X
Y (. . 6.12):
ma = mg sin F ,
(6.84)
0 = mg cos N .
(6.85)
(6.84) (6.85) (6.83) (6.82),

:
7
tg .
(6.86)
2
5
7
: a = g sin tg .
7
2
6.5

m1 m2. F.
1 2
?

I ,
X Y ,
. 6.13. F .

, .

6.

211

m2g
F

F
. 6.13

II. (
) X Y :
m2 a2 = F ,
(6.87)
0 = N m2 g ,
(6.88)
F , , N , .
1 X
F ,
F
, .

X :
m1a1 = F F .
(6.89)
(6.47)
,
:
J 0 = F R ,
(6.90)
d
dt
. (6.90) , ,
.

(6.87) (6.90) ,
:
a2 = a1 R .
(6.91)
J 0 , =

212

, , (6.45) :
2
J 0 = mR 2 .
(6.92)
5
III. (6.87), (6.89) (6.92),
:
7F
(6.93)
a1 =
,
7 m1 + 2m2
2F
.
(6.94)
7 m1 + 2m2
, .
,
(. . 2.1.2 2)
:
F N .
(6.95)
a2 =

.

, X Y
(6.87) (6.88) (6.95). (6.94) :
7 m + 2m2
F g 1
.
(6.96)
2
:
7 m + 2m2
7F
2F
a2 =
F g 1
.
a1 =
2
7 m1 + 2m2
7 m1 + 2m2
6.6
, , ,
, . 6.14. ,
.

. m1 m2, M
R, a1 a2, -

6.

213

.
Y
.

T1
T3
I. T4
T3
,
, X Y T1
Mg
T2

. 6.14. T2

X,
m1g
X
m2g

(m1g, m2g, Mg) (T1, T2, T3 T4).
. 6.14
,
.
II. X :
m1a1 = m1 g T1 ,
(6.97)
m2 a2 = m2 g T2 ,
(6.98)
Ma2 = Mg + T2 T3 T4 .
(6.99)

, (. . 6.14), :
(6.100)
J 0 1 = T1R T3 R ,
J 0 2 = T4 R T3 R ,
(6.101)
1 2 , J0 (6.44):
MR 2
J0 =
.
(6.102)
2
,
:

214

a1 + 2a2 = 0 ,
(6.103)
a1 = 1R ,
(6.104)
1 = 2 .
(6.105)

: a1 , a 2 , T1 , T2 , T3 , T4 , 1 , 2 J 0 .
III. a1,
(6.97), (6.98) (6.100) (6.105):
T1 = m1 g m1a1 ,
(6.106)
m
T2 = m2 g 2 a1 ,
(6.107)
2
M

(6.108)
T3 = m1 g m1 + a1 ,
2

T4 = m1 g (m1 + M )a1 .
(6.109)
,
:
M
T1 T3 = T3 T4 =
a1 .
(6.110)
2
(6.106) (6.109) (6.99) X:
4m 2m2 2 M
a1 = 1
g.
(6.111)
4m1 + m2 + 4 M
(6.103),
X:
m + M 2m1
a2 = 2
g.
(6.112)
4m1 + m2 + 4M
(6.110)
(6.111) :
2m1 m2 M
T1 T3 = T3 T4 =
Mg .
(6.113)
4m1 + m2 + 4M
6.7
, . 6.15,
m1, m2, J0 R1 R2 (R2 > R1).

6.

215

. a1
a2 .

I. , T0
(. . 6.15), Y .
T A
T m2g
, Y, - Y
m1g

. ,
. 6.15
, ,
A
.
,
. 6.15.
II. Y (.
. 6.15) :
m1a1 = m1 g T ,
(6.114)
T , .
( )
:
J = m2 gR T ( R2 R1 ) .
(6.115)
J ,
. (6.114) , T0 , (. 6.15), .
J0 -
(6.42):
J = J 0 + m2 R12 .
(6.116)

216

(6.114) (6.116) , :
a1 = ( R2 R1 ) ,
(6.117)
a2 = R1 .
(6.118)
III. (6.114) (6.118), a1 a2:
(m1 (R2 R1 ) m2 R1 )g (R R ) ,
a1 =
(6.119)
2
1
2
m1 (R2 R1 ) + m2 R12 + J 0
a2 =

(m1 (R2 R1 ) m2 R1 )g R .
1
2
m1 (R2 R1 ) + m2 R12 + J 0

(6.120)

, (.
(6.119) (6.120)),
,
.
m
R1
, a1
1 >
m2 R2 R1
(6.119),
R1
a2 = a1
, (
R2 R1

R2 < 2 R1 ), ( R2 > 2 R1 ) a1 .
,
.
,
R2 = R1
a1 , a2 =
J 0 =

a2 =

2
g.
3

m2 R12
g . m2 R12 + J 0

1
m2 R12
2

6.

217

6.8

m = 100 J0 = 400 2
.
R = 4 , r = 1 .
= 60 F = 0.2
(. . 6.16).

X
F
R

r
mg
d N
F Z

. 6.16

a ,
, .

I. ,
X, Y Z
, . 6.16. , , , .

. Z , (. 6.16).
II.

218

, X Y :
d
J
= Fd ,
(6.121)
dt
0 = N mg + F sin .
(6.122)
ma = F cos F ,
(6.123)
J
, , d
F
( F), F ,
, N .
(6.121) (6.123) (
), - (6.42) J (. . 6.16):
d
a=
R,
(6.124)
dt
J = J 0 + mR 2 .
(6.125)
d
r
+
=R.
(6.126)
cos cos
, ,
(. . 2.1.2 2) :
F N .
(6.127)
III. (6.121) (6.126)
a, :
FR 2
r
a=
cos .
(6.128)
2
R
J 0 + mR
:

6.

219

F
J cos + mrR
0
.
(6.129)
mg F sin
J 0 + mR 2
.
(6.128) a
r

cos . , R

F Y ,
r
, cos > 0 .
R
.
, ,
:
a = 0.4 /2, 0,075.

6.9
.
(.
. 6.17). R, m1, m2.
, .

Y
N2
T

N1
T
F1
X

m1g
. 6.17

F2
m2g

220

I. ,
, X Y . 6.17.
m1 g m2 g ,
F1 F2 , N1 N 2 ,

T (. . 6.17).
II.
X Y :
m1a = m1 g sin T F1 ,
(6.130)
m2 a = m2 g sin + T F 2 ,

(6.131)

0 = N1 m1 g cos ,
(6.132)
0 = N 2 m2 g cos ,
(6.133)
(6.130) (6.131) , T, , .
,
X .
a , .
, , (. (6.47)):
(6.134)
J1 = F1R ,
J 2 = F 2 R .

(6.135)

J1 J 2 , ; ,
,
:
R = a .
(6.136)

(6.44)
, :
m R2
J1 = 1 ,
(6.137)
2

6.

221

J 2 = m2 R 2 .
(6.138)
(. . 2.1.2
2) , , :
F1 N1 .
(6.139)
F2 N 2 .

(6.140)

III. (6.130) (6.138),



, ,
:
N1 = m1 g cos ,
(6.141)
N 2 = m2 g cos ,
(6.142)
m + m2
F1 = 1
m1 g sin ,
(6.143)
3m1 + 4m2

2(m1 + m2 )
m2 g sin .
(6.144)
3m1 + 4m2
(6.141) (6.144) (6.139)
(6.140), , :
3m + 4m2
,
(6.145)
tg 1
m1 + m2
F2 =

3m1 + 4m2
.
(6.146)
2(m1 + m2 )
(6.146)
(6.145)
m1 m2,
, , (6.146).
(6.146)
:
3m1 / m2 + 4
.
arctg
(6.147)
2(m1 / m2 + 1)
tg

222

. 6.18 tg / m1 / m2 .
tg /
2
1
0
0

10

m1 / m2
. 6.18

, ,
, , arctg(2 ) m1 << m2

3
arctg 2
m1 / m2 (. 6.18).
6.10
r = 3
, R = 10 . 0 = 30 /,
.
= 0,1 . ,
, t, .

I. , . n

6.

223

(. . 1.1. 1),
(. . 6.19).

N
F . , n
(t )
, .
N

, F
0
, ,

. . 6.19

,
.
II.
n :
man = N ,
(6.148)
ma = F .
(6.149)

(. . 1.1.
1):

an =

,
(6.150)
Rr
d
a =
,
(6.151)
dt
( R r )
.
(6.47) , , :
d
J0
= F r .
(6.152)
dt
(6.44) :

224

mr 2
.
(6.153)
2
( t < t ) (.
. 2.1.2. 2) , :
F = N .
(6.154)
J0 =


( t t ):
= r .
(6.155)
III. (6.148) (6.151)
(6.154),
( t < t ):

d
2
=
.
(6.156)
dt
Rr
(6.156) :

R r 0

dt ,

(6.157)

Rr
.
(6.158)
R r + 0 t
(6.148) (6.154):
d
2
=
2 ,
(6.159)
(
)
dt r R r
(6.158) (6.159), :

(t ) = 0

(t ) =

2 02 (R r )
1
dt .
2

r
0 (R r + 0 t )
t

(6.160)


:
202 t
(6.161)
(t ) =
.
r (R r + 0 t )

6.

225

(6.158)
0 ( t = 0),
(6.161). ( t 0 )
(6.157).
. 6.20

r .

, r, /c
30

20

10
0
0

t, c

. 6.20

t (. . 6.20). (6.158) (6.161) (6.157) t:


Rr
t =
.
(6.162)
2 0
:
2
(t = t ) = 0 .
(6.163)
3
,
0 ,
.
(6.162) (6.163),
:
t 1,66 , = 20 /.

226

6.4.
1
1 2 XY. R r (.
.), .

1
r

X
y ,
.

R + 2r
(R + r)
, = 1 2 ,= 1
.
: y = 2
R+r
1 2
(R + r)
2
h r
(. .). O , .
.
O
h

6.

: =

227

r
1+ , =
.
rh
r
h
2

3
R
.
. n = 2
.
3 R
: =
.
8 g
4
,
R r,
(. .).
.
( ) .
(R + r)
R
, =
.
: =
r
r
5
R
(. .). .
,
.
a .
2 g
4
: a = g , = .
5 R
5

r
R

228

6
m
(. .).

a , .
1
1
: a = g sin , F = mg sin .
2
2
7
.
(. .).

, m.
F .
mg sin
.
: F =
7
8
m R
.

.
, -

6.

229

.
.
: x =

2R2
.
18g

9
m1 m2 , m (.
.).
m1
m

m2


.
.
, .
m2 m1
, m2 > m1 ;
: a = g
m
m1 + m2 +
2
a = 0 , m2 m1 .
10
M
(. .). L
m. a
x.
2mgx
.
: a =
ML + 2m( L x)

230

11
,
, .
,
.
. m,
M R, a. .
m+M
: a =
g.
7
m+ M
2

M
M

7.

231

7

. .

7.1.
( ) 1 L , M ex
:
dL
= M ex = 0 dL = 0 .
(7.1)
dt
( ) Ln

, M nex :
dLn
= M nex = 0
dt

dLn = 0 .
(7.2)

(7.1) (7.2) :
L L(t2 ) L(t1 ) = 0 L(t1 ) = L(t2 ) ,
(7.3)
Ln Ln (t2 ) Ln (t1 ) = 0 Ln (t1 ) = Ln (t2 ) .
(7.4)

(.
(6.38) (6.39) . 6.1 6).

( ) (), ( )
() . 6.1 6.

232

2
:
2
1
1
E k = mi i2 = mi (V + [ri]) =
2 i
2 i

1
2
mi V 2 + 2V [ri] + [ri] =

2 i

1
1
] + mi [ri]2 .
mV 2 + mV [r
2
2 i

(7.5)

mi i ,
, V S', ,
S', ri -
- (. 3) S', r
S'.
S',
, ,
,
:
1 2 1
2
E k = m
(7.6)
+ mi [ri] ,
2
2 i
.
:
1
E k = J 2 ,
(7.7)
2
J , .


. 3.1 3, . 6.1 6.

7.

233

:
2

A = M n d , A12 = M n d ,

(7.8)

Mn (. . 6.1
6), 1 2
.

:
1
1
] + J 2 .
E k = mV 2 + mV [r
(7.9)
2
2
, (7.5)
(7.7).
S',
, ,
,
, ( ):
1
1
2
E k = m
+ J 2 ,
(7.10)
2
2
, J
, .
, J n
3:
1
E k = J n 2 .
(7.11)
2
, :
2
1
1
E k mi i2 = mi [ri ] ,
(7.12)
2 i
2 i

. 6.1 6.

234

ri - ,
, . 2
2
[ab] = a 2 b 2 (ab ) , :
2
1
1
2
E k = mi [ri ] = mi 2 ri2 (ri ) =
2 i
2 i

1
2 r 2 x =
m

i
i i

2 i

1
= mi ri2 xi xi =

2 i
,

1
1
= mi ri2 xi xi = J ,
2 , i
2 ,

Ek =

1
J .
2 ,

J = mi ri2 xi xi

(7.13)

, -

;
.
:
1
E k = J n 2 ,
(7.14)
2
J n .
, .
,
3 .

7.

235

Fi np,in Fi np,ex 4:

dE = Anp,in + Anp, ex = Anp ,


(7.15)

E = Anp .
(7.16)

,
:
E E (t2 ) E (t1 ) = 0

E (t1 ) = E (t2 ) .
(7.17)

- ,
(.
. 7.1).
dt

dL

M
mg

. 7.1.


(.
. 7.1).

,
. 3.1
3.

236

:
- ;
-
.

L
L = J z ,
(7.18)
(6.38)
O (. . 7.1) :
dL
=M,
(7.19)
dt
Jz , M ( ), .
(7.19) L
.
. 7.1 , :
dL = L sin dt ,
(7.20)
dL = [L]dt .
(7.21)
, :
dL
= [L] .
(7.22)
dt
(7.19) :
M = [L] = J z [] .
(7.23)
, ,
, ..
!
,
(, , ..)
( ) .

7.

237


:
M = M = [L] = J z [] .
(7.24)
.. .
7.2.
7.2.1.

, "
. . "
.
1)
( ),
2) .
7.2.2.
I. .
1. , .
2. ( ), () ,
, (), .
3. .
4. ( ) .
5. ( )
-

238

( , , ..).
6.
() .
II. .
1. ()

.
2. ,

.
3. .
4.
.
III. .
1. .
2. ( , ,
).
3. .
7.3.
7.3.1.

7.1
, m l, M L l,
(. . 7.2). .
max c
.

7.

239

I. m
/2 l
,
. - M, L
(. . 7.2).
:
. 7.2
t1 , t2
, t3 , t4 , .
(t1, t2)
. (t2, t3)

+ . , , . (t3, t4) .
.
II. (7.17)
(t1, t2):
2
J11
mgl =
.
(7.25)
2
mgl 2

J11
2
( t2), J1
, 1 .
(t2, t3)
(7.4) (7.17) + :
J11 = J 22 + J13 ,
(7.26)
( t1),

240
2

2
2
J
J11
J
(7.27)
= 2 2 + 1 3 ,
2
2
2
J2 , 2 3 .

(7.17) (t3, t4):
2
J 22
MgL
(1 cos max ) .
=
(7.28)
2
2
J 1 J 2
:
J1 = ml 2 ,
(7.29)
1
J 2 = ML 2 .
(7.30)
3
III. (7.25) (7.30) , :

L
24
max = arccos1

.
(7.32)
2

2
l

3 + M L

m
l

max / 2 , M / m L / l
:
24
L
<1,
(7.33)
2
2

l
M
L

3 +

m l

.
. 7.3
, / 2 . , . 7.3 l / L m / M ,
, . -

7.

241

, , .

l/L
1
0.8

max = / 2

0.6
0.4

max < / 2

0.2
0
0

0.4

0.8 1.2
m/M

1.6

. 7.3

. 7.4
max (l / L )
(m / M ) .

max ,
90

m/M = 2

0,5

60

0,3
0,1

30
0
0

0.2

0.4

0.6

0.8

l/L
. 7.4

,
, -

242


.
. 7.5
max (m / M )
(l / L) .

max ,
90

l/L = 1

0,3

0,5

60

0,2

30

0,1

0
0

0.4

0.8 1.2
m/M

1.6

. 7.5

,
,
.

L
24

.
: max = arccos1

2 2 l

3 + M L

m
l

7.2

m L
M.
(. . 7.6).
B l
( O). :

7.

243

1)
Y
,
A
2) ,
3) (
A . 7.6)
O
X
.

B
I. .
. 7.6
,
+
,
: ,
. , . 7.6. ,
,
.
II.
.
X
:
m = m + M .
(7.34)
:
ml = m l + J 0 .
(7.35)
:
m 2 m 2 M 2 J 0 2
=
+
+
.
(7.36)
2
2
2
2
,
X ( Y ), .

244

, , (6.43):
1
(7.37)
J 0 = ML 2 .
12
(.
(1.26) 1) A

, :
L
A = .
(7.38)
2
III. (7.34) (7.38)
:
(m M ) J 0 + Mml 2
=
,
(7.39)
(m + M ) J 0 + Mml 2
m
2 MJ 0
=
,
(7.40)
M (m + M ) J 0 + Mml 2
ml
2 MJ 0
=
,
(7.41)
J 0 (m + M ) J 0 + Mml 2

1
2mMJ 0
lL

.
(7.42)
2
(m + M ) J 0 + Mml M 2 J 0
l ,
, (7.39)
= 0 (7.37):
M m
.
(7.43)
l=L
12m
(7.40), l = 0. .
l, A
, (7.42) (7.37):
1
l = L.
(7.44)
6

A =

7.

245

,
A.
:
M m
1) , l = L
;
12m
2) ,
(l = 0);
3) A
1
l = L .
6
7.3
m r
0 . , 1 2 ,

1 = 2 = 0 . ,
.

E k .

I. . ,

. X
, .
II. X ,
:
m0 m0 = 2m ,
(7.45)
X
2m. ,
= 0 , ,
.

246


, :
L1 + L2 = L ,
(7.46)
L1 L2 , L . , ,
(6.27) 6
.


(6.32) 6 :
L1 = J 0 1 ,
(7.47)
L2 = J 02 ,
(7.48)
L = J ,
(7.49)
J 0 J
. (6.45):
2
J 0 = mr 2 ,
(7.50)
5
2
J = ( 2m ) R 2 .
(7.51)
5
R (, , ):
4
4
2 r 3 = R 3 .
(7.52)
3
3
:
1 = 2 = 0 .
(7.53)

E k (7.6) :

7.

247

m0 2 J 00 2
J 2
.
2
+
(7.54)
2
2
2
III. (7.46) (7.53), :
3

2
J
= 20 0 cos = 0
cos .
(7.55)
2
2
2
J
(7.46) (7.49)
J
= 0 (1 + 2 ) ,
(7.56)
J

(7.53), ,
1 2 .

, (7.55)
(7.54) (7.50) (7.52):

J
2
E k = J 002 0 (1 + cos ) 1 m0 =

J
E k =

2 2 2 3 2
(1 + cos ) 1 m 0 2 .
mr 0
5

(7.57)

7.4
m , ,
D

2
1 2 ,
C
. 7.7.
A
, 1
, J, B
l.
. 7.7
?

248

I. : , X ,
. 7.8, ,
, X', . 7.9.
, , , , ,
, , .

C
A

1
B

C
X

X'
u

B
. 7.8

. 7.9

, , , .
II. 1 2, ,
X
(. 3)

= 1 2 ,
(7.58)
2
u1 u2 X' :
+
u1 = 1 = 1 2 = u ,
(7.59)
2
+
u2 = 2 = 1 2 = u .
(7.60)
2

7.

249

X'
,
, :
mu1 + mu2 = mu1 + mu 2 ,
(7.61)
u1 u2 D
.
2
u2

C
A
X'
u1
- 1
,
B
, . 7.10

(. . 7.10).

, , , ,
(7.10):
mu12 mu22 mu1 2 mu22 J12 J12
+
=
+
+
+
.
(7.62)
2
2
2
2
2
2
1 2 .

,
,
, :
l
l
l
l
mu1 + mu2 = mu1 + J1 mu 2 + J2 .
(7.63)
2
2
2
2
(7.63) ,
(6.23) 6

, ,
, .
, -

250

(. . 7.11):
1 = 2 = ,
(7.64)
u2 = u1 = u .
(7.65)
D
u

C
A

X'

B
. 7.11

III. (7.59) (7.65)

u1 , u2 , 1 2 :
u1 = u2 =

1 + 2 ml 2 4 J

(7.66)
2
= u ,
2
ml + 4 J
4lm
+
1 = 2 = 1 2 2
= .
(7.67)
2
ml + 4 J
, m
, :
2

ml
l
J 2 m =
.
(7.68)
4
2
, (7.66) u = u1 = u2 0 (.
. 7.11).
:
ml 2 4 2 J
,
(7.69)
1 = u + = 1 2
ml + 4 J
ml 2 41 J
.
(7.70)
2 = u + = 2 2
ml + 4 J
, ,

7.

251

,
(. . 7.12).
D

2
C
A

B
. 7.12

, ,
, ,
:
ml 2
(7.71)
J=
.
4
:

1 = 2 = 1 2 ,
(7.72)
2
+
= 1 2.
(7.73)
l
(7.65) .
, , , (. 8 3).
. 7.5
l0 m0 = 10
,
, (.
. 7.13). , ,
.

252


0 = 60.
m
m

1 = 45 , ?

I. l0, m0
+
.
. 7.13
,
, ,
.

: (l = 0, = 0 ) (l = l0, = 1 ). : l
.
l, , .

, .
+ ,
.

,
( & 0 ), ,
(. (4.16) 4)
: F F . (. (4.17)
4), , : F 0 .

7.

253

,
+ , , .
, ,
,
. ,
, , ,
. ,
, .
,
+ ,
, ,
.
II. + L L L . L
. ,
. + :
l0

L = L + L = x sin
0

m0
dxx sin + l sin ml sin =
l0

2
(7.74)
= sin 2 m0l0 + ml 2 ,
3

x dx .

+

254

(l = 0, = 0 )
(l = l0, = 1 ):
1
1
2
L1 L0 = 1 sin 2 1 (m0 + 3m )l02 0 sin 2 0 m0l0 = 0 ,
(7.75)
3
3
L0 0 O

,
F

L1 1.
mg
, dF
+ , dl
,
m0g
O (.
. 7.14
. 7.14):

M
+ M
+ M
+ M
= 0.
(7.76)

, M

M
, :

M
= m0 gl0 sin ,
2

M = mgl sin .

(7.77)
(7.78)

(. (4.16) 4),
M
, :

M
= m 2l 2 sin cos .
(7.79)
,
, dx,
M
x . (. (4.16)), ,

m
dF = 0 dx 2 x sin ,
(7.80)
l0
x sin
.

7.

255

, ,
,
O, (. . 7.21):
m

dM
= dF x cos = 0 dx 2 x 2 sin cos .
(7.81)
l0
(7.104) , , :
l0

m0
1
dx 2 x 2 sin cos = m0 2l02 sin cos .
l
3
0 0

M
=

(7.82)

III. ( )
.
(7.76) (7.79) (7.82), :
3g (m0l0 + 2ml )
=
(7.83)
.
2 m0l02 + 3ml 2 cos

(7.83) (l = 0, = 0 ) (l = l0, = 1 ) :

0 =

3g
,
2l0 cos 0

(7.84)

1 =

3 g (m0 + 2m )
.
2l0 cos 1 (m0 + 3m )

(7.85)


+ (7.75), :
1 sin 2 1 (m0 + 3m ) 0 sin 2 0 m0 = 0 .
(7.86)
(7.84) (7.85) (7.86) m:
sin 4 0 cos 1
2
=0.
6m 2 + 5m0 m + m0 1

(7.87)
4

sin 1 cos 0
(7.87), :
1
sin 4 cos 1
m = 5 + 1 + 24 4 0
m0 .
12
sin 1 cos 0

(7.88)

256

, ,
m 0,32 m0 = 3,2 .
7.6
m
, (. . 7.15).
1

r2

r1

. 7.15

r1 r2
, 1 .
M,
R1 R2 .

I. , , .
,
. (. . 2.1.2. 2) . (. 3.1.2. 3),
,
,
(. . 7.1. ) .
, +
,

7.

257

(. 3.1.2. 3). (. 7.1. ).


II.
, (. . 7.15),
r1
r2 :
r2 m 2 r1m1 = 0 ,
(7.89)
2
. (7.89) , r . ,
r / 2 (. 7.15).

+ r1 r2 :
m 22
m12
+ E2p
E1p = 0 .
(7.90)
2
2
E1p E2p + .
r .
,
. (. (3.32) 3)
:

mM
mM
d r = G
.
(7.91)
2
r
r
r
, +
r1 r2 :
mM
,
(7.92)
E1P = G
r1
E = G
p

258

mM
.
(7.93)
r2
,

, :
mM
2
m 1 =G 2 ,
(7.94)
R1
r1
E2P = G

22

mM
.
(7.95)
R2
r22
III. . II
(7.89) (7.95)
r
2
M = 1 (r2 + r1 ) 1
(7.96)
2G
r2
m

=G


rr
R1 = R2 = 2 1 2 .
(7.97)
r1 + r2
,
, ,
.
7.3.2. .
7.7
,
, l.
.
,

,
J. m.

7.

259

I.
. Z Z

. , ,
F1
F2
(. . 7.16).

,
,
. 7.16
,
.
. , . ,
, , .
II. :
dL
=M,
(7.98)
dt
L .

, M
, .
:
l
l
M + F1 F2 = 0 ,
(7.99)
2
2
F1 + F2 mg = 0 .
(7.100)
F1 F2 , (. . 7.16).
, ,
(7.101)
L = J .

260

:
d
.
(7.102)
=
dt
, .
, .
III. (7.98) (7.102) F1 F2, :
mg J
F1 =
+
,
(7.103)
2
l
mg J
F2 =

.
(7.104)
l
2
mgl
,

2J
.
7.8
() m, ,

(. . 7.17).
m, J0

. 7.17

r. J0.
.

7.

261

I. , . 7.18.
Z AA' 1. . 7.18. C.
Z

A'

mg
Y

1
N

X
. 7.18

mg, N
,
( . 7.18). , , .
, , R . , , .

, N,
.
II. C
M mg M N . C :
dL
= M N + M mg .
(7.105)
dt

262

,
1
Lxy
. LXY (. . 7.19) -
Z (. . 7.20), LZ Lz
L
:
. 7.19
dL = dLXY + dLZ = dLXY .
(7.106)

Lxy(t)
Z

Lxy(t)

dLxy

Lxy(t+dt)
Lxy(t+dt)
. 7.20

M mg
M N C Y (. . 7.18). , LXY (7.105) (7.106) :
dLXY
(7.107)
= M N + M mg = RN Rmg .
dt
LXY
AA', (6.30) 6:
LXY = J 01 .
(7.108)

, , , , .

7.

263

, ,

.
1 :
R 1r = 0 .
(7.109)
LXY . . 7.20.
:
dLXY LXY d
(7.110)
=
= LXY .
dt
dt
(7.107)
(7.110) N.
III. (7.107) (7.110)
, :
R
dLXY
(7.111)
= J 0 2 = R(N mg ) ,
dt
r
J
N = 0 2 + mg .
(7.112)
r

N, (7.112).
7.9

R = 5 m0, ,
OO' (. 7.21).
OO' CD.

OO' m0.
OO' , n = 50 ./.
m0

, m0

m0

. 7.21

264

m = m0 / 10 .
CD, l CD
m0 (. . 7.21) 10 .

I. m ,
,
, m0
OO' CD . . m
,
. , L .
II. (. (7.23)) OO' CD (.
. 7.18):
M = [L] ,
(7.113)
M , ,
.
(7.18) OO':
L = J 0 .
(7.114)
, , ,
(6.44) 6
m R2
J0 = 0 ,
(7.115)
2

n :
= 2n .
(7.116)
m0 m
CD
,
F
F = (m0 + m )g .
(7.117)

7.

265

, m0, m0 m
OO' CD AB (. . 7.22)
M = m0 gl + (m0 + m )gl = mgl .
(7.118)

B
L
O'

A
D
. 7.22


OO' (. 7.22).
(7.113) CD (. 7.22).
(7.114) (7.118) (7.113)
M , L , :
M
mgl
=
.
(7.119)
=
L nm0 R 2
CD (7.118) :
m R2
2
T=
= 2 2 n 0 .
(7.120)
mgl

, ,
T = 12,5 c .

266

7.4.
1
, 1 , ,
, 2 (. .). 1
J1 J2.
J1
,
,
2

J2
. A, ,
.
1 JJ
: A = 1 2 (1 2 ) 2 .
2 J1 + J 2
2
, , (. .).

h0 0, - h
0
.
h0
h ?
?
8 gh
2
: h = 0 1 + 1 + 2 0 ; 4 g
0
8 gh

: = 0 1 + 2 0 .
2
0
3
l m . m0 = m / 8 , -

7.

267

0,
l0 = l / 4
. .
4
: = 0 .
13 l
4
,
, l0 m (. .).
0 = l0 , l0
. -
0
M
,
m
?

?
2
: M (t ) = 2ml0 2e 2t , l (t ) = l0 et .
5
= 40 / R = 300 .
M,
, J0 = 3,6103 2 n = 150 ./. .

: M = 2nJ 0

= 2,1103 .

6
m = 0,5
= 200 /. J = 5 10-4 2.
= 0,5 /.

= 30 0 . -

268

l .
J
: l =
= 0,5 , = sin = 50 /2.
mg
7

.

, .

.
.
2 R
: T =

.
4 g

mg

8
m, ,
(. .).
J, l.

.
mgl
: =
.
J
9
A
l L.

.
. M m

A
M, L

l
m

7.

269

?
M
l2
=3 2 .
:
m
L
10
m
F = kr .
-
r = r0 ,
.
max .

max =

k
r0 .
m

11

1 2 , -

1 2 (. .).
,
. m
R ,
E k .

1 2

m
6(1 + 2 ) 2 + 6 R 2 (12 + 22 ) R 2 (1 + 2 ) 2 ,
24
1 + 2
.
=
6

: E k =

270

8

.
8.1.

. , (. . 6.1.1 6),
(. . 8.1).

(t )

. 8.1. (t )

, . ,
(t ) , (. . 8.2).

(t )

. 8.2. (t )

T ,
(. 8.2).

8.

271

, (t ) (.
. 8.3).

(t )

t
. 8.3. (t )

() ,
( ).
8.1.1.
, ,
:
&& + 02 = 0 ,
(8.1)
, ; 0 2
T0 =
-

, .
( )
:
(t ) = A cos(0t + 0 ) .
(8.2)
(0t + 0 ) ; A 0 ,
0 (t = t0 ) &0 &(t = t0 )
t0:

272
2

&
A = + 0 ,
0
2
0

(8.3)

&0
.
(8.4)

0 0
& ( ):
&(t ) = A0 sin (0 t + 0 ) .
(8.5)
,
0 , /2.
:
1) ,
2) .
, .
, (. . 8.4).
m
k

0 = 0 t0 arctg

. 8.4.

. X , , ,

8.

273

,
(. 8.4).
F
, (.
. 2.1. 2).
X :
m&x& = kx ,
(8.6)
m , k .
(8.6) :
k
&x& + x = 0 .
(8.7)
m
(8.7) (8.1), :
k
0 =
.
(8.8)
m
,
. ,
(8.7)

.
, , (8.2) (8.5) :
x(t ) = A cos(0t + 0 ) ,
(8.9)
x& (t ) = A0 sin(0 t + 0 ) .
(8.10)
, :
mx& 2 (t ) mA 202
E k (t ) =
=
sin 2 (0 t + 0 ) =
2
2
2
kA
=
sin 2 (0t + 0 ) .
(8.11)
2
, , :
kx 2 (t ) kA2
E p (t ) =
=
cos 2 (0t + 0 ) .
(8.12)
2
2

274


20 (. . 8.5).
, , :
kA2
E = E k (t ) + E p (t ) =
.
(8.13)
2

Ek,p(t)

Ek(t)

T/2

Ep(t)

. 8.5. E Ep

, (.
. 8.6).

,
l
.
F
, , . mg
(6.39)
,
. 8.6.

(. . 8.6):
dL
= M mg ,
(8.14)
dt

8.

275

L = l ( ml& ) = ml 2& , , m l , M mg = mgl sin ,


.
(8.14) (8.1):
ml 2&& = mgl sin ,
(8.15)
g
&& + = 0 .
(8.16)
l
(8.16) (8.1), :
g
0 =
.
(8.17)
l

(8.2) (8.5) :
(t ) = A cos(0t + 0 ) ,
(8.18)
& (t ) = A0 sin(0 t + 0 ) .
(8.19)
, :
ml 2& 2 mglA2
Ek =
=
sin 2 (0 t + 0 ) .
(8.20)
2
2

.
,
:
mglA2
E p = mgl (1 cos )
cos 2 (0t + 0 ) .
(8.21)
2
, ,
20 (. . 8.5). :

276

mglA2
(8.22)
.
2
, (. . 8.7).

,


l
J0
.

mg
, , . . 8.7.
(6.48)
, (. . 8.7):
(8.23)
J&& = M mg .
E = Ek + Ep =

, J

, M mg = mgl sin ,
, m l
.
(8.23) (8.1):
J&& = mgl sin .
(8.24)
mgl
&& +
=0.
(8.25)
J
(8.25) (8.1), :
mgl
.
(8.26)
0 =
J
(6.42),
J 0 , :

8.

277

mgl
.
(8.27)
J 0 + ml 2
, .
:
(t ) = A cos(0t + 0 ) ,
(8.28)
& (t ) = A0 sin(0 t + 0 ) .
(8.29)
(. (7.7)
. 7.1. 7):
J& 2 mglA2
Ek =
=
sin 2 (0 t + 0 ) .
(8.30)
2
2
, :
mglA2
E p = mgl (1 cos )
cos 2 (0t + 0 ) .
(8.31)
2
:
mglA2
E = Ek + Ep =
.
(8.32)
2

0 =

8.1.2.
:
&&(t ) + 2& + 02 = 0 ,
(8.33)
(
).
(8.33) .


( < 0).
(. . 8.8):

278

(t ) = Ae t cos(t + 0 ) .
(8.34)
2
2
= 02 2 T =
=

02 2
.

(t )

. 8.8. (t )

t
t + T:
(t )
(8.35)
ln
= T .
(t + T )
,
(t )
ln
= NT = N .
(8.36)
(t + NT )

, e 2.7 :
1
(t )
ln
= N e = 1 , = N e .
(8.37)

(t + N eT )

,
E p E k :
E

= Ek

+ Ep

= E0 e 2 t .

(8.38)
T

8.

279

dET
dE
=
= 2 E0e 2 t .
(8.39)
dt T
dt
Q
:
ET

E0e 2 t
= 2
=
= =
.
(8.40)
Q 2
2 t

T 2
P TT
2E0e T
P


( > 0).
:

(t ) = A1e

+ 2 02 t

2 02 t

+ A2e
,
(8.41)
A1 A2 , .

A1 A2 , .
A
1 > 0 (t ) A2
t (. . 8.9).

(t )

. 8.9. (t )
A
1 > 0
A2

A1
< 0 (t ) A2
,

280

t (.
. 8.10).

(t )

. 8.10. (t ) A
1 < 0
A2

( = 0).
:
(t ) = ( A1 + A2t )e t ,
(8.42)
A1 A2 , .


. 8.11.

(t )

. 8.11. (t )


0
(t ) t .

8.

281

8.1.3. .

:
&& + 2& + 02 = B cos( pt ) ,
(8.43)
B cos( pt ) , B p .
, , , .
< 0 .
:
(t ) = (t ) + (t ) = (t ) + A( p) cos( pt + ( p) ) .
(8.44)
(t ) , (t ) , A(p) (p)
(t ) ,
(.
. 8.12 8.13):
B
A( p ) =
,
(8.45)
2
02 p 2 + 4 2 p 2

2p
.
(8.46)
p 02
. 8.12 . 8.13
2.
t >> 1/,
(t ) :
(t ) = (t ) = A( p ) cos( pt + ( p) ) .
(8.47)
tg ( p) =

282

A(p)

2
A
0

p = 02 2 2

. 8.12. A(p)
p

(p)

/2

. 8.13. (p)
p

( )
A( p)
(. 8.12).
p d A( p)

=0:
dp

p = 02 2 2 .

(8.48)

:
B
A = A( p ) =
.
(8.49)
2 02 2

8.

283

( p = 0 )
:
B
A = A(0) = 2 .
(8.50)
0
( p >> 0 )
(. 8.12):
B
p
A( p ) ~ 2
0 .
(8.51)
p
,
A A . (8.40), (8.49)
(8.50):
Q=

( 0 >> ).
2
A

(8.52)


:
&(t ) = & (t ) = A( p) p sin ( pt + ( p ) ) =
= A( p ) p cos( pt + ( p ) + / 2 ) .
(8.53)
A( p ) p (.
. 8.14):
Bp
.
(8.54)
A( p ) p =
2
2 2
2 2
0 p + 4 p

. 8.14 2.
A( p ) p &(t )
(. 8.14).

d ( A( p) p )
= 0 (8.54) :

dp
p = 0 .
(8.55)

284

A(p)p

p = 0

. 8.14.
A(p)p p

( p = 0 )
&(t ) (. 8.14):

( Ap ) = 0 .

(8.56)
( p >> 0 ) :
B p
(8.57)
A( p ) p ~
0 .
p
8.2.
8.2.1.

" . " . :
1) ,
2) ,
3) , .
.
, .

8.

285

8.2.2.

,

, 2 ( ) 3
( ). , 6
( ) 7 ( ). , . .
I.
.
II. .
III. .
8.3.
8.1
( )
m R ,

k
k
,

, m, R
k
(. 8.15).
.
. 8.15
.

I.
, .
X .

286

X . .
(. . 8.16):
mg,
2F
2F ,

.
. , .
II. (.
mg
(6.48) 6)
0
X
(. 8.16),

. 8.16

:
(8.58)
J&& = mgR sin 2kx ( 2 R ) mgR 4kxR .
J ,
(. 8.16), . (8.58) ,
,
2 R , sin .
,
:
x = 2 R .
(8.59)
,
,
(. (6.42) 6):
mR 2
3
J=
+ mR 2 = mR 2 .
(8.60)
2
2
III. (8.59) (8.60) (8.58),
:
2 8k g
(8.61)
&& + = 0 .
3 m R
,

8.

287

2 8k g
(8.62)
.
3 m R

8k g
8k g
> .
,

m R
m R
(.
. 8.1.1 ).

0 =

8.2
( )

R , , (. 8.17).

O
N1

N2

mg
. 8.17

.
, ,

0 .

I.
, . .
mg
N1 N2 (. 8.17).
.

288

II. (. (6.48) 6)
, ,

(. 8.17):
J&& = mgl sin ,
(8.63)
J ,
, l
. (8.63) ,

N1 N2 .
, :
R 3
l = R sin( / 3) =
.
(8.64)
2
(6.42):
J = J 0 + ml 2 .
(8.65)
, , :
mR 2
J0 =
.
(8.66)
12
III. (8.63) (8.66)

(sin ) , :
3 3g
= 0.
(8.67)
5R
(. (8.1)),

&& +

3 3g
,
(8.68)
5R
( (8.67)) :
(t ) = A cos(0 t + 0 ) .
(8.69)
A 0

:

0 =

8.

289

(0) = 0 = A cos 0 , & (0) = 0 = A sin 0 .

(8.70)
(8.70) :
A = 0 , 0 = 0 .
(8.71)

:
3 3g
(t ) = 0 cos
(8.72)
t .
5R

8.3
( )
, ,
l m.
. V0 , . . ,
Am AM
.

O
I. :

, ,
, . F mg

, (. . 8.18),
. 8.18
.
V0

, O,
-

290

.
. (. 8.18)
mg, F . .
II. , , (4.16) . 4.1. 4 :
F = m&x&M ,
(8.73)
&x&M ( ) .
(
; . (6.48) . 6.1.2 6)
, O (. 8.18):
ml 2&& = mgl sin m&x&M l cos ,
(8.74)
(. . 8.18).
(8.74) ,
ml 2 , .
xm (t ) :
(8.75)
xm (t ) = xM (t ) + l sin (t ) ,
x M (t ) ,
, .
(,
&&(t ) &x&M (t ) )
,
+ .
x (. (3.1)
. 3.1. 3) :
mx + MxM
.
(8.76)
x = m
m+M

8.

291


( ), , :
&x& = 0 .
(8.77)
X
.
(8.76) (8.75)
(8.77),
:
( m + M ) &x&M + ml & 2 sin + && cos = 0 .
(8.78)
( sin )
(8.74), (8.75) (8.78) :
(8.79)
ml 2&& = mgl m&x&M l ,
(8.80)
xm (t ) = xM (t ) + l (t ) ,
(8.81)
( m + M ) &x&M + ml&& = 0 .
(8.79) (8.81) x m (t ) x M (t ) .
III. (8.79) (8.81),
(. (8.1)
. 8.1.1. ) :
g
&& + (1 + m / M ) = 0 .
(8.82)
l
, 0
:
g
(1 + m / M ) .
0 =
(8.83)
l
. 8.19 0 m / M (
l = 2 ). , , m << M -

292


g
(8.17) 0 =
.
l

0 , c 1
8
6
4
2
0
0

10

15

m/M
. 8.19

(8.82) :
(t ) = A cos(0t + 0 ) .
(8.84)
A 0 (8.84) , :
(t = 0) = 0 ,
(8.85)
V
& (t = 0) = 0 .
(8.86)
l
:
V
V
(t ) = 0 cos(0t + / 2) = 0 sin(0t ) .
(8.87)
l 0
l 0
(8.87) &&
(8.81)
x M :
m/M
&x&M =
V00 sin(0 t ) .
(8.88)
1+ m / M
(8.88)
xM (t = 0) = 0 ,
(8.89)
x& M (t = 0) = V0 ,
(8.90)

8.

293

:
1
xM (t ) =
V0t + AM sin(0t ) ,
(8.91)
1+ m / M

m / M V0

.
(8.92)
AM =
1 + m / M 0
. 8.20
x M (t )
m / M .
(8.91)
V0 = 1 / l = 2 .
,
1

V0
1+ m / M
0 AM (.
(8.91)).
xM ,

m/ M 0

xm ,

0,25

m/ M 0

0,25

4
3

4
3

10

4
10

0
0

4
t, c
. 8.20

4
t, c
. 8.21

294

, m << M
, AM .
x m (t ) ,
(8.90) (t ) (8.87) x M (t )
(8.91):
1
xm (t ) =
V0t Am sin(0t ) ,
(8.93)
1+ m / M

V
1
Am =
0 .
(8.94)
1 + m / M 0
. 8.21
xm (t ) m / M
. (8.93)
, (.
. 8.20). , , ,

1

V0 1+ m / M
0 , Am (. (8.94)). , m >> M ,
Am .
. 8.22
AM Am . , Am
m / M . AM , , .

8.

295

A,
Am (8.94)
0.4
Am
l
max
Am = V0

g
AM
AMmax

0
m/ M 0.
0 2 5
10
15
m/M
AM (8.92)
. 8.22

m / M = 2 l
2
V0
.
AMmax =
g
3 3
8.4
( )
r ,
J ( ) m
R , (. 8.23). .

O
V

mg

N
R
F
. 8.23

296

I. mg ,
N F

(.
. 8.23). . , .
. ,
, , , , (. . 3.1.3
3).
II.
, V ,
,
(. (7.10) . 7.1. 7):
mV 2 J 2
Ek =
+
.
(8.95)
2
2
J , , ,
. 8.22
V = ( R r )& ,
(8.96)
, .

,
(. . 8.23)
mg
E p = mg ( R r )(1 cos )
( R r ) 2 .
(8.97)
2
,
k
E + Ep = 0 .
(8.98)
t
,
. , ,

8.

297


:
V = r .
(8.99)
(8.96) (8.99)
,
,
, & :
Rr
=
& .
(8.100)
r
III. (8.95) (8.98) (8.100), :
mgr 2
&& +
= 0.
(8.101)
( R r )( mr 2 + J )
(8.101) (8.1),
:

0 =

mgr 2
.
( R r )(mr 2 + J )

(8.102)

mr 2

, J =
2

2 2

J = mr :
5

2g
0 =
,
(8.103)
3( R r )

0 =

5g
.
7( R r )

(8.104)

8.5
( )
0
U- , . -

298

S s ,
L.

I. , . . . , ,
,
. : A1 , A2
,
.
II. S(s).
L

m = S ds (L ).
0

,
. , S1 S 2 ,
A1S1 = A2 S 2 .
(8.105)
s .
s = L . A
0 s L S A1S1 = AS ,
(8.105). V
s
S
V = 0 A = 0 A1 1 .
(8.106)
S

:

8.
L

k
=
Emax

S ds V 2
2

299

02 A12 S12 d s
2

(8.107)


A + A2
A1S1 = A2 S 2 1
:
2
A + A2
S1 A12 g S1
p
1 + .
= S1 A1 1
Emax
g=
(8.108)
2
2
S2

, :
k
p
Emax
= Emax
.
(8.109)
III. (8.107) (8.108) (8.109),
0 :
L

S1 A12 g

S
1 + 1 .
(8.110)
2
S
2
S2
0
(8.110)
:
g (S1 + S 2 )
0 =
.
(8.111)
L
ds
S1S 2
S
0

02 A12 S12 d s

(8.111) S = const
U- :
2g
0 =
.
(8.112)
L
8.6
( )

,
. R r . J . , .

300

m .
k , ,
(. 8.24). .
,
V0 .

R
r

m
k

I. . 8.24
. X
(.
. 8.25). m mg, FA, T1
, F = x& (. (2.12)

. 2.1.2. 2).
.
II.
:
(8.113)
m&x& = mg F T1 x& .

,


(. 8.25):
J&& = T1r T2 R .
(8.114)
, T1 T2
,
.
, ,
, :
T2 = k ( x x,0 ) ,

T2

T1
T1

F
x

X
mg

. 8.25

(8.115)

8.

301

x , x,0
.
,
:
x
,
(8.116)
=
r
x
.
(8.117)
=
R
(8.116) , :
&x&
&& = .
(8.118)
r
(8.116)
(8.117), :
R
x = x .
(8.119)
r
(8.119), (8.115) :
R
(8.120)
T2 = k ( x x0 ) ,
r
x0 , .
(8.113),
(8.114), (8.118) (8.120), .
III. , T1 T2 (8.113),
(8.114), (8.118) (8.120),
x :
J
R2

&
&
&
(8.121)
m
x
x
mg
F
k
x
x
+
+

+
+
(

)
=0.

0
r2
r2

x ,
( x& = 0 &x& = 0 ):
r2
x = x0 + (mg F ) 2 .
kR

(8.122)

302

, , :
= x x .
(8.123)
(8.121)
:

r 2

kR 2
= 0,
(8.124)
mr 2 + J
mr 2 + J
(. (8.33)
. 8.1. ).
(8.33),
0
:
r2
=
,
(8.125)
2(mr 2 + J )

&& +

& +

kR 2
.
mr 2 + J
(8.124)
(t ) = e t cos( t + 0 ) ,

0 =

(8.126)
(8.127)

= 02 2 ,
, A 0 , .
:
x(t ) = x + e t cos( t + 0 ) ,
(8.128)

, ,
x(t = 0) = x ,

(8.129)

x& (t = 0) = V0
(8.130)
A 0 :
V0
A=
,
(8.131)
02 2

8.

0 =

303

.
(8.132)
2
(. . 8.26):
V0
x(t ) = x +
e t sin 02 2 t .
(8.133)
02 2

x (t )

. 8.26

,
, < 0 (. . 8.1.2. ). ,
(8.121) (8.41)
+ 2 2 t
0

x(t ) = x + A1e

2 2 t
0

+ A2 e

(8.134)

1 2
(8.129) (8.130):
V0
A1 = A2 =
.
(8.135)
2 2 02
:
2 2 t
+ 2 02 t
0
V0

x(t ) = x +
e
. (8.136)
2
2
2 0

(8.136) (.
. 8.27), , .

304

x(t )

. 8.27

8.7
( )
m R, ,
0 .
(. . 8.28). ,
,
f = , = const ,
.
. 8.28
M = D , D -

, . .

I. .
.
: ,
, .
, .
II. (. (6.48) . 6.1.2. 6) ,
:
J&& = M + M ,
(8.137)

8.

305

J , M
.
.
,
, (. (6.44) 6):
mR 2
.
(8.138)
J=
2
dM , r
dS = 2rdr:
dM = 2rdrr = 2r 3dr& .
(8.139)
, , ,
:
R

M = 2 2& r 3dr = R 4& .

(8.140)

III.
(8.140) (8.137) (8.138) :
2R 2
2D
&& +
& +
=0.
(8.141)
m
mR 2
(8.141)
(8.33), 0 :

R 2

,
(8.142)
m
2D
0 =
.
(8.143)
mR 2
( < 0 )
(8.141) (. (8.34)):
= Ae t cos(t + 0 ) ,
(8.144)
= 02 2
, A , 0 .
, ,

306

(t = 0) = 0 ,
& (t = 0) = 0

(8.145)
(8.146)
A 0 :
A = 0 ,
(8.147)
0 = 0 .
(8.148)
(.
. 8.29):

(t ) = 0 e t cos 02 2 t ,

(8.149)

(t )
0
t

. 8.29

, < 0 (. . 8.1.2. ). ,
(8.141) (8.41)
+ 2 2 t
0

(t ) = A1e

2 2 t
0

+ A2 e
,
(8.150)
A1 A2
(8.145) (8.146):

.
A1 = 0 1 +
(8.151)
2
2
2

.
A2 = 0 1
(8.152)
2
2

8.

+ 2 02 t
e

+
2
2
2

2 02 t

+ 0 1
.
(8.153)
2
2
2

(8.153) (.
. 8.30), , .

(t ) =

307

1+

(t )
0
t

. 8.30

8.8
( , )
m = 100 , k = 40 /,
F = F0 cos pt ,
p = 25 / F0 = 1 .
= 3 / 4 . Q ,
p , -

, A .

I. , :
, , F = F0 cos p t . . 8.1.1, , .

308

,
. , ,
.
II. (8.40):
Q=

.
2

(8.154)

, . 8.1.2 :

= 02 2 .

(8.155)

0
(. (8.8)) m k:
k
.
(8.156)
0 =
m
, (8.154)
(8.155),
(8.46):
2p
tg = 2
.
(8.157)
p 02
(8.48) :
p = 02 2 2 .

(8.158)


(. (8.49)) :
F0
A = A( p ) =
.
(8.159)
2m 02 2
(8.154) (8.159) Q , p A .
III. (8.154) (8.157) :

8.

Q=

02 p 2

tg 2 ( p 2 02 ) 2

309

1
1
kmp 2
=
.
2
2
2
4
4
tg (mp k )

(8.160)

p ,
(8.156) (8.158):
p

tg 2 ( p 2 02 ) 2
k tg 2 mp 2 k
=
=

m
2 p2
2m 2p 2
2
0

(8.161)

A ,
(8.156), (8.157) (8.159):
F0 p
A = A( p ) =
=
tg 2 ( p 2 02 ) 2
2
2
2
m( p 0 ) tg 0
4 p2

F0 p

k tg 2 mp 2 k
(mp 2 k ) tg

m
4m 2 p 2

(8.162)


(8.160) (8.162), :
Q 2,17 ; p 19,0 / ; A 5,7 .
8.9
( , )

F (t ) = F0 cos(pt) . , 0 . F(t)
, ,
.

I. F (t ) = F0 cos(pt ) , (. . 8.1.3).

310

II. (. (8.47) (8.53)):


x(t ) = a cos( p t + ) ,
(8.163)
(t ) = ap sin( p t + ) .
(8.164)
dA F,
:
dA = F (t )dx = F (t ) (t ) dt ,
(8.165)
:
F (t ) = F0 cos(pt) .
(8.166)
T :
T

A = F (t ) (t )dt .

(8.167)

:
T

P =

1
F (t ) (t )dt .
T 0

(8.168)

(8.163) (8.168) P
p.
F , ,
, (8.45), (8.46) (8.48) a( p ) , ( p)
p :
a( p) =

F0

m 02 p 2

+ 4 2 p 2

(8.169)

2p
.
p 02

(8.170)

p = 02 2 2 .

(8.171)

tg ( p ) =

III. (8.168) (8.167)


F(t), :

8.

311

1
F0 a( p) p sin ( ( p ) ) .
(8.172)
2
sin( ( p ) ) , (8.172), (8.170):
tg ( p)
2p
(8.173)
sin ( p ) =
=
.
2
2
1 + tg ( p )
02 p 2 + 4 2 p 2
P =

(8.173) (8.169) (8.172),


:
F02 p 2
.
(8.174)
P ( p ) =
2
m 02 p 2 + 4 2 p 2

p max ,
dP ( p )
,
=0:
dp
pmax = 0 .
(8.175)
, pmax , (. . 8.1.3).
(8.175) (8.174) :
F2
Pmax P ( pmax ) = 0 .
(8.176)
4m

, , (8.171) (8.174):
F 2 2 2 2
P P ( p ) = 0 0 2
.
(8.177)
4m 0 2
F(t) , , , (8.176) (8.177):
P 02 2 2
=
.
(8.178)
Pmax 02 2

)
)

312

, <

2
2
( ).

8.10
(
)
m
l , . k
2
1
(. . 8.31).
, a F2
F1
. mg
mg
1 (t )

. 8.31
2 (t )
:
1)
0 t = 0 ;
2) 0 t = 0 ;
3) t = 0

0 , .

I.
: mg , Fi
(i = 1, 2) , . 8.31. .

8.

313

, ,
,
(. 8.31).
, .
II. (. (6.48) . 6.1.2. 6) ,

(. . 8.31):
ml 2&&1 = mgl sin 1 F1a cos 1 ,
(8.179)
ml 2&&2 = mgl sin 2 + F2 a cos 2 .
(8.180)


. (8.179)
(8.180) , .
, , (. (2.5) . 2.1.2
2), :
F1 = ka(sin 1 sin 2 ) .
(8.181)
,
, ,
, , , ,
( ):
F1 = F2 .
(8.182)
(8.181), (8.182) (8.179), (8.180) i ( sin i i ,
cos i 1 ), :
g
ka 2
&&1 + 1 + 2 (1 2 ) = 0 ,
(8.183)
l
ml
g
ka 2
&&2 + 2 + 2 ( 2 1 ) = 0 .
(8.184)
l
ml

1 = 1 + 2 ,
(8.185)
2 = 1 2 ,
(8.186)

314


(8.1) 1 2 :
g
&&1 + 1 = 0 ,
(8.187)
l
g
2ka 2
= 0 .
&&2 + +
(8.188)
2 2
l 1 ml
, 1
2 :
1 (t ) = A1 cos(1t + 1 ) ,
(8.189)
2 (t ) = A2 cos(2t + 2 ) ,
(8.190)
1 2 (8.187) (8.188)
:
g
1 =
,
(8.191)
l
g 2ka 2
.
(8.192)
+
l
ml 2
A1 , A2 1 , 2 1 2 .

(8.185), (8.186), (8.189) (8.190) 1 (t )
2 (t ) :
A
A
1 (t ) = 1 cos(1t + 1 ) + 2 cos(2t + 2 ) ,
(8.193)
2
2
A
A
2 (t ) = 1 cos(1t + 1 ) 2 cos(2t + 2 ) .
(8.194)
2
2
A1 , A2 1 , 2 .
,
(8.193) (8.194) :
A
A
&1 (t ) = 1 1 sin(1t + 1 ) 2 2 sin(2 t + 2 ) ,
(8.195)
2
2

2 =

8.

315

A11
A
sin(1t + 1 ) + 2 2 sin(2 t + 2 ) .
(8.196)
2
2
1. ,
0
t = 0 , :
1 (t = 0) = 0 , 2 (t = 0) = 0 ,
(8.197)
&1 (t = 0) = 0 , & 2 (t = 0) = 0 .
(8.198)
(8.193) (8.196) A1 , A2 1 , 2 :
A1 = 2 0 , A2 = 0 ,
(8.199)
1 = 0 , 2 = 0 .
(8.200)

:
g
1 (t ) = 0 cos
t,
(8.201)
l

& 2 (t ) =

g
t.
(8.202)
l
, g
1 =
, 0 .
l
2. ,
0 t = 0 ,
:
1 (t = 0) = 0 , 2 (t = 0) = 0 ,
(8.203)
&1 (t = 0) = 0 , & 2 (t = 0) = 0 .
(8.204)
,
(8.193) (8.196) A1 , A2 1 , 2 :
A1 = 0 , A2 = 2 0 .
(8.205)
1 = 0 , 2 = 0 .
(8.206)
:

2 (t ) = 0 cos

316

g 2ka 2
+
t,
l
ml 2

1 = 0 cos

(8.207)

g 2ka 2
(8.208)
t.
+
l
ml 2
,

2 = 0 cos

g 2ka 2
+
0 .
l
ml 2
3. , t = 0
0 , ,
:
1 (t = 0) = 0 , 2 (t = 0) = 0 ,
(8.209)
2 =

, & 2 (t = 0) = 0 .
(8.210)
l
, (8.193) (8.196) A1 , A2 1 ,
2 :

&1 (t = 0) =

A1 =

1 =

0
gl

, A2 =

, 2 =

0
2ka 2
gl +
m

(8.211)

.
(8.212)
2
2

:

1 =

2 =

0
2 gl

0
2 gl

sin

g
t+
l

sin

g
t
l

0
2ka 2
2 gl +
m

0
2ka 2
2 gl +
m

sin

g 2ka 2
+
t,
l
ml 2

(8.213)

sin

g 2ka 2
+
t.
l
ml 2

(8.214)

8.

317

,
g
g 2ka 2
2 =
.
+
l
l
ml 2
:
1 = C1 sin 1t + C2 sin 2t ,
(8.215)
2 = C1 sin 1t C2 sin 2t ,
(8.216)

1 =

C1 =

2 gl

, C2 =

2ka 2
2 gl +
m

(8.217)

mgl
1
2
2 (. (8.191) (8.192))
2 1 << 1 ,
(8.218)
(8.216) (8.217) :
1 (t ) = C1 (sin 1t + sin 2t ) + (C2 C1 ) sin 2t

ka 2 <<

+ 2 1 2
2C1 sin 1
t cos
t =
2
2

= 2C1 cos t sin t ,


2

2 (t ) = C1 (sin 1t sin 2t ) (C2 C1 ) sin 2t

(8.219)

+ 2 1 2
2C1 cos 1
t sin
t =
2
2

(8.220)
= 2C1 sin t cos t ,
2


+ 2
= 1
, = 2 1 .
(8.221)
2
,
-

318

= .
T =

2
,
.
. 8.32

1(t)

2
= 1,1 .
1

2(t)

. 8.32

. 8.33

2
= 9 ,
1

.
,
, (
).

8.

319

(. . 9.1. 9).

1(t)

2(t)

. 8.33

( ).
8.11
(
)
m, l0 k, . F, . ,
l(t).

I. F
(. . 8.34). , . X

320

+
(. 8.34).

F
O

x1

x2

F
X

. 8.34


, (.
. 2.1. 2).
, (
) , , . X :
m&x&1 = k ( x 2 x1 l0 ) ,
(8.222)
m&x&2 = F k ( x 2 x1 l0 ) ,
(8.223)
x2 x1 ( ) .
(8.222) (8.223)

,
l x :
l (t ) = x2 (t ) x1 (t ) ,
(8.224)
x (t ) + x1 (t )
x (t ) = 2
.
(8.225)
2
(8.222) (8.223),

m&l& = F 2k (l l0 ) .
(8.226)
l (t ) z (t ) :
F
l (t ) = z (t ) + l0 +
.
(8.227)
2k
(8.226)
(8.1):
2k
&z& +
z=0.
(8.228)
m

8.

:
z = A cos(0t + 0 ) ,
0

321

(8.229)

2k
,
(8.230)
m
A 0 , :
l
l
x1 (t = 0) = 0 , x2 (t = 0) = 0 ,
(8.231)
2
2
x&1 (t = 0) = 0 , x& 2 (t = 0) = 0 .
(8.232)
(8.224), (8.227),
(8.231) (8.232) :
F
(1 cos 0 t ) + l0 .
(8.233)
l (t ) =
2k
. 8.35
l (t )

l(t). , F
- l0 +
k

,

l0
l0 ( )
t
0
T0
F
. 8.35
l0 + .
k
(8.222) (8.223)
(8.225)
:
F
.
(8.234)
&x& =
2m
(8.231),
(8.232) :
Ft 2
.
(8.235)
x (t ) =
4m

0 =

322

l (t ) x (t ) (8.224) (8.225) (8.233) (8.235) :


l (t ) F 2
l
(8.236)
x1 (t ) = x (t )
=
t 1 + cos(0t ) 0 ,
2
4m
2
l (t ) F 2
l
x2 (t ) = x (t ) +
=
t + 1 cos(0t ) + 0 .
(8.237)
2
4m
2
. 8.36 x (t )

x 2 (t )
. , x1 (t )

F

&x& =
l0 / 2
2m

0
2k
0 =
, - l / 2
0
m

.
t
0
T0
, , . 8.36
F
,
,
l0
F
l0 .
k

8.4.
1
. . h , . ,

8.

323

,
S.

: A =

m m
2mh

.
+
S S M + m

2
,
.
, ,
, . k. .
l .
A A. .
M
m
k (m + M )
: =
, A = l
, A = l
.
+
+
m
M
m
M
mM
3
R
R/2 ,
. , ,
.
29 R
: T =
.
g
4
m
,
.
k
k (. .). m
O
, .

324

: T = 2

m
.
3k

5

m l , (. .). k ,
. .
3 g 3k
: =
+
.
2l m

O
m
k

6
R
R/2 , .
, , .
177 R
: T = 2
.
10 g
7
Q
l = 50 , = 5,2
= 4104 .

g
1,3 10 2 .
: Q
ln l
8
M z = M 0 cos t ,
, = 0 cos(t ) .
, , .
: = 0 0 sin .

8.

325

9
1 2
. , .
: = 12 .
10
.
2
: = 1
, = 12 (1 2 ) 2 / 12 .
2 3

326

9
.

9.1.
,

( (t , r ) (t , r ) ), .
.
(t , r ) t - r , , .
(t , r ) t - r , , .

.
, (t , r )
.
() (, , , ) . , ,
.
9.1.1.
,
:
2 (t , r )
= c 2 (t , r ) ,
(9.1)
t 2
(t , r ) , ,

2
2
2
+
+
.
x 2 y 2 z 2

9. .

327

,
:
2 (t , r )
= c 2 (t , r ) ,
(9.2)
t 2
(t , r ) ( ).
() (9.1) (9.2).
( )
:
(t , r ) = i (t , r ) , (t , r ) = i (t , r ) .
(9.3)
i

, i (t , r ) (
i (t , r ) )
(9.1) ( (9.2)), ,
.
,
, . ,
, ,
.
,
(
(t , r ) (t , r ) ).
9.1.2.

(t , r )
, , x X
(t , x ) . ,
.
:
2
2 (t , x)
2 (t , x )
&&(t , x ) = c 2 x' ' (t , x ) ,
(9.4)
c
=
t 2
x 2
.

328

(9.4)
( )
:
x
x
(9.5)
(t , x) = f1 t + f 2 t + .
c
c
x
f1 t , c
x
X, f 2 t + ,
c
.
n
(. . 9.1).

. 9.1.
r n

, - r ,
x = n r
nr
(9.6)
(t , r ) = t
.
c


.

(t , x ) YZ x = 0
():
(9.7)
(t ,0) = 0 cos(t + 0 ) ,

9. .

329

0
YZ x = 0, ,
1
2
T = = 2 =
; 0

T
( x = 0 t = 0).
(9.5)
x 0 .
:
(t , x ) = 0 cos ( (t x / c ) + 0 ) = 0 cos (t kx + 0 ) =
= 0 cos(t + ( x) ) = 0 cos( (t , x) ) .
(9.8)
2 2
, , kx
k = =
=

c
c
( ) x, (t , x) = t kx + 0 () .
, r , :
(t , r ) = 0 cos(t kn r + 0 ) = 0 cos(t k r + 0 ) ,
(9.9)
k kn , k
( ).
( )
, (t , r )
.
:
(t , r ) = 0e r cos(t k r + 0 ) ,
(9.10)
.
9.1.3.

(t , r )

S. -

330

S,
.
( (9.2)
):
(t r / c) 2 (t + r / c)
,
(9.11)
(t , r ) = 1
+
r
r
1 (t r / c) 2 (t + r / c) , r x 2 + y 2 + z 2 r S,
(. . 9.2).

Z r
0
S
r
Y
X

. 9.2.
r0 r

(t r / c )
. 1

r
(t + r / c)

, 2
r
, .
r0
, :
(t , r0 ) = 0 ( r0 ) cos( (t r0 / c) + 0 ) .
(9.12)
r0 .
:

9. .

331

A
cos(t kr + 0 ) ,
(9.13)
r
A , S.
(
) :
A
(t , r ) = 0 (r ) cos(t kr + 0 ) = e r cos(t kr + 0 ) . (9.14)
r

(t , r ) = 0 (r ) cos( (t r / c) + 0 ) =

9.1.4.

.
:
E
c|| =
,

(9.15)


(), , .
, E

():
= E .
(9.16)
dx
x
(. . 9.3).

(x)

(x+dx)

(x)

(x+dx)

x+dx

. 9.3. (x )

332


( x + dx) ( x)
=
=
= x' ,
(9.17)
x
dx

( x) = E x' .
(9.18)
S
, , dm = Sdx
:
Sdx&& = S ( ( x + dx ) ( x ) ) .
(9.19)
(9.19) (9.18)
:

&& =
= x' ,
x
E
&& = x'' .
(9.20)


(9.4), (9.15)
.
.
:
G
,
c =

(9.21)

, ()
, = tg . , -

E > G
c|| > c .
:
= G .
(9.22)

,
x x + dx
(. . 9.4).

9. .

333


(9.22) . 9.4 :

( x) = G ( x) = G tg( ( x) ) = G
,
(9.23)
x x

( x + dx) = G ( x + dx) = G tg ( ( x + dx) ) = G

(9.24)

x + dx

(x+dx)

(x+dx)
(x)

(x)
x x + dx

. 9.4. (x )
X


dm = Sdx :
Sdx&& = ( ( x + dx ) ( x ) )S .
(9.25)
(9.25) (9.23) (9.24):
G
&& = x'' .
(9.26)


(9.4), (9.21)
.
.
:
T
,
c=

(9.27)

T , .

334


dx dm = dx (.
. 9.5)
:
( x + dx )
( x )
dx&& = T ( x + dx )
T ( x)
.
(9.28)
x
x

T(x+dx)

(x+dx)
(x)
T(x)

x x+dx

. 9.5. (x )

(9.28)

T(x) = const:
( x ) 2
( x )
,
dx && = T
+ 2 dx T
x
x
x

&& =

x'' .

(9.29)


(9.4), (9.27)
.
.
:

c=

P
,

(9.30)

P , 0 .

9. .

335

( ) :

c=

P
P
= 0 ,
0

(9.31)

cP / cV , P cV
, P0 .
dx ( )
x (. . 9.6).

(x)

(x+dx)

P(x)
x

P(x+dx)
x+dx

. 9.6. (x )


:
P
Sdx&& = S (P( x ) P( x + dx ) ) = S
dx ,
x
P
&& =
.
(9.32)
x
P = P ( ) .
P ,
, :
P
P =
.
(9.33)

0


(. . 9.6):

336

= x' .
(9.34)
dx

x

:
P
(P ) P

P
2
&& =
=
=

=
2 ,
x
x
x
x
x
0
0

P
&& =
x'' .
(9.35)

=

( x + dx) ( x)


(9.4), (9.30)
.

dV = Sdx dm = dV:
P(dV ) = const C , P = C (dV )

dm

= C

P
P

= C (dm ) 1 = 0 .
0

= C (dm ) .

(9.36)

(9.31)
.
9.1.5.
X
(t , x ) , &(t , x ) (t , x ) (. . 9.1.2
9.1.4.):
(t , x) = 0 cos(t kx + 0 ) ,
(9.37)
&(t , x ) = sin(t kx + ) ,
(9.38)
0

(t , x) =

= 0 k sin (t kx + 0 ) .
x

(9.39)

9. .

337

, , :
& 2 (t , x ) 02 2 2
w k (t , x ) =
=
sin (t kx + 0 ) .
(9.40)
2
2
,
:
E 2 02 Ek 2 2
wp (t , x) =
=
=
sin (t kx + 0 ) .
(9.41)
2
2
2

E

(. (9.15)) k , :
c =

c
wp (t , x) =

02 Ek 2
2

sin 2 (t kx + 0 ) =

2
2
0

sin 2 (t kx + 0 ) = wk (t , x) .
(9.42)
2
, .
:
w(t , x) = wk (t , x) + wp (t , x) = w0 sin 2 (t kx + 0 ) ,
(9.43)

w0 = 02 2 = 02 Ek 2 .
(9.44)
:
w 2 2
w(t , x) T = 0 = 0
.
(9.45)
2
2
,
, ,
. :
w(t , x)cs d t
S (t , x )
= w(t , x)c ,
(9.46)
sdt
s .
=

338

,
,
. :
S (t , x ) w(t , x )c .
(9.47)

:
w
I S ( x, t ) T = w( x, t ) T c = 0 c .
(9.48)
2
,
w0 = 02 2 (9.44),
(9.45) (9.48), .
,
.
9.1.6. ()
s d , , , c
,
(. . 9.7).

cTs

sTs

cTd

dTd
X

. 9.7.

. 9.7 , ,
, ,
t. t + Ts (Ts -

9. .

339

) cTs, sTs . ,
(. 9.7):
= cTs sTs .
(9.49)
t' . ,
, t' + Td
(Td ). Td dTd , cTd,
+ dTd = cTd .
(9.50)
, Ts = 1/s Td = 1/d, (9.49) (9.50)
s d:
c d
vs .
vd =
(9.51)
c s
9.1.7.
() ,
( ).
,
L > c ,
(9.52)
c , .
() . () , . .
.
( ).

340


, .

.
,
.
, .
()
. .

(9.3) , c 0 , (, ):
x
x
(t , x) = 1 t + 2 t + =
c
c
= 0 cos(t kx + 01 ) + 0 cos(t + kx + 02 ) =

01
+ 02

= 2 0 cos kx + 02
cos t + 01

2
2

C cos(kx + 0 ) cos(t + 0 ) ,
(9.53)
01 02 ( t = 0)
01
+ 01
0 = 02
, C cos(kx + 0 )
x = 0 , C = 2 0 , 0 = 02
2
2
.
(9.53) ,
(. . 9.8), :
kx + 0 = n , n = 1, 2, 3, ... .
(9.54)

9. .

341

(. 9.8) :
2n + 1
.
(9.55)
kx + 0 =
2
, , .

(t , x )

t=0
t = T/6
x
t = T/3
t = T/2

. 9.8.

(t , x ) ,
:
= &(t , x ) = C cos(kx + 0 )sin(t + 0 ) =
= 0 cos(kx + 0 )sin(t + 0 ) ,
(9.56)

= x' = Ck sin (kx + 0 ) cos(t + 0 ) =


= 0 sin (kx + 0 ) cos(t + 0 ) .

(9.57)
( (9.56) (9.57) (9.53)),
,
, .
/4.
.

wk (t , x) wp (t , x)

342

, , wk wp :
T

(t , x)
2

1
= C 2 2 cos 2 (kx + 0 )sin 2 (t + 0 ) =
2
2
k
2
= w0 cos (kx + 0 ) sin 2 (t + 0 ) ,
(9.58)

wk (t , x) =

w0k
1
(9.59)
cos 2 (kx + 0 ) = C 2 2 cos 2 (kx + 0 ) ,
T
4
2
E 2 1 2 2 2
wp (t , x) =
=
= C Ek sin (kx + 0 ) cos 2 (t + 0 ) =
2
2
2
p
2
= w0 sin (kx + 0 ) cos 2 (t + 0 ) ,
(9.60)
wk

wp

1
w0p
sin 2 (kx + 0 ) = C 2 Ek 2 sin 2 (kx + 0 ) ,
2
4

(9.61)

1
1
w0k = w0p = C 2 2 = C 2 Ek 2 .
(9.62)
2
2
:
1
1
w T = wk + wp = C 2 2 = C 2 Ek 2 .
(9.63)
T
T
4
4
(9.58) (9.60) . ,
,
,
, ,
, .
( ,
, )
.
(
), ,

w k .
, ,
p w p
.

9. .

343

( , ),
, , p
w p . , w k .
n (), , 2c
.
c, : n =
n

L
.
(9.53).
,
:
(t , x = 0) = C cos( 0 ) cos(t + 0 ) = 0 ,
(9.64)
(t , x = L) = C cos(kL + 0 ) cos(t + 0 ) = 0 ,
(9.65)
(9.64) (9.65) t, :
cos( 0 ) = 0 ;
(9.66)
cos(kL + 0 ) = 0 .
(9.67)
(9.67) (9.66), :
cos(kL + 0 ) = cos(kL ) cos( 0 ) sin (kL )sin ( 0 ) = sin (kL ) = 0 .
:
2 n
2
L = n ,
L = n , n = 1, 2, 3, .
k n L = n ,
c
n
, () :
2L
n =
,
(9.68)
n

344

c
.
(9.69)
2L
,
""
:

n = n

L=n

.
(9.70)
2

( ) .
. 9.1.

L=n

L = (2n + 1)

2L
n

n = n

4L
2n + 1

n = (2n + 1)

n =

n
4

n =

c
2L

L=n

n
2

n =

2L
n

n = n

L = nn

n =

L
n

n = n

L = ( 2n + 1)

2L
2n + 1

n = ( 2n + 1)

n
2

n =

c
4L

c
2L

c
L
c
2L

9.2.
9.2.1.

" .
" . :
1) ;
2) ;

9. .

345

3) , ,
.
, ,
.
9.2.2.
I. .
1. , .
2. ( ).
3.
.
4. , ( ).
II. .
1. ():
) ,
) ,
) ,
) .
2. .
3. , .
4.
(,
).
III. .
1. .
2. ( , ,
).
3. .

346

. II.1, II.2
.
II.1 II.3 .
9.3.
9.1

0 = 1 = 1

= 1,3 /3 c = 340 / ,
= 60 = 45 X Y
.
x1 = 1 , y1 = 1 , z1 = 1 x2 = 6 ,
y2 = 6 , z2 = 6 , ,
= 60 c
s = 10 2 , Z.

I.
, X, Y, Z .
II.
(. (9.9)):
(t , r ) = 0 cos(t k r + 0 ) ,
(9.71)
r - .
k
c ,
k r = kx cos + ky cos + kz cos ,
(9.72)

x, y, z , k =

cos = 1 cos 2 cos 2 .


(9.73)
, , , X, Y, Z,
:

9. .

347

= 0 cos(t kx cos ky cos kz cos + 0 ) .

(9.74)
x1, y1, z1 x2,
y2, z2 :
1 = t k r1 = t kx1 cos ky1 cos kz1 cos + 0
(9.75)

2 = t k r2 = t kx2 cos ky2 cos kz2 cos + 0 . (9.76)


(. . 9.1.5)
,
s , Z, :
(9.77)
E = S z T s .
Z
:
S z T = S (t , r ) e z T = S (t , r ) T e z = S (t , r ) T cos .
(9.78)

,
(9.47):
S (t , r ) T w(t , r ) T c .
(9.79)
(. (9.45)) :
w(t , r )

02 2

.
(9.80)
2
(9.73), (9.75),
(9.76) (9.73), (9.77) (9.80)
= 2 1 E .
III. ,
:
=

(( x2 x1) cos + ( y2 y1) cos +

+ ( z 2 z1) 1 cos 2 cos 2 .


E=

02 2cs

1 cos 2 cos 2 .

(9.81)

(9.82)
2

:

348

/ 2 ,
E = 2,6 10 4 .
9.2
= 2 103 /3 X

(t , x ) = 0 cos(t kx ) . 0 = 1 / , c = 2 / . ,
, ,
(t = 0, x = 0) = 0 .

I.
, X
.
II. :

(t , x) = (t , x)dt =

0
sin(t kx) + C ( x) ,

(9.83)

C (x ) (9.83)
x = 0:
(t = 0, x = 0) = 0 .
(9.84)
(9.83) (9.84) ,
C ( x ) = 0
:

(t , x) = 0 sin(t kx) = 0 sin(t kx) ,


(9.85)

0 =

0
.

(9.17) (9.83) :
(t , x)
(t , x) =
= 0 k cos(t kx) = 0 cos(t kx) ,
(9.86)
x
0 = 0 k .

9. .

349

(9.48) (9.44):
w
2 2c
.
(9.87)
I = 0c= 0
2
2
III. (9.85) (9.86) 0 =

0
:

0
sin(t kx) ,
(9.88)

(t , x) = 0 k cos(t kx) ,
(9.89)

,
, (9.87):
2 c
(9.90)
I= 0 .
2
(9.90)
, , :
I = 200 / 2 .
(t , x) =

9.3
= 1,45 . r0 = 5
0 (r0 ) = 50 , P,
r = 10 , 0 (r )
= 3 0 (r0 ) .
0 (r ) P.

I. , . , , , .

350

II. (. (9.14)):
A
(t , r ) = e r cos(2t kr + 0 ) .
(9.91)
r

:
(t , r )
A
(t , r ) =
= 2 e r sin (2t kr + 0 ) =
t
r
= 0 (r )sin (2t kr + 0 ) ,
(9.92)
0 (r )
A
0 (r ) = 2 e r .
(9.93)
r
r0
A
0 (r0 ) = e r0 ,
(9.94)
r0
P r :
A
0 (r ) = e r = 0 .
(9.95)
r
III. (9.94) (9.95), :
r
= ln 0 (r r0 ) .
(9.96)
r

P, (9.93) (9.94) (9.95):
A
(r )
(9.97)
0 (r ) = 2 e r = 2 0 0 .
r

(9.96) (9.97)
, , :
0,08 -1 ,
0 (r ) 15 / .

9. .

351

9.4
P0 = 10 .
J, .
, , s = 4 2. = 1,3 /3,
c = 334 /.

I. . , X
. .
II. J, s,
(9.48) :
(9.98)
J = s w(t , x ) T c .

(9.45),
:
w(t , x)

02 2

(9.99)
.
2
,
0 P0 , .
(9.8):
(t , x) = 0 cos(t kx + 0 ) .
(9.100)
(9.34)
, ,

= x ,
(9.101)
T

x , (9.100):
x = 0 k sin(t kx + 0 ) .
(9.102)

(. (9.33) (9.36)):

352

(9.103)
=
,

P
P .
(9.31) :
P
(9.104)
c2 = .

III. (9.101) (9.104), , :


(9.105)
P = c 2 0 k sin(t kx + 0 ) .
, :
P0 = c 2 0 k = c 0 .
(9.106)
J, s,
(9.98) (9.99) (9.106):
2 2
P 2 2
P02
=
.
(9.107)
J = s w(t , x) T c = s 0
c=s 0
c
s
2
2
2c
2(c )
(9.107) , , :
J = 4.6105 .
9.5
S ,
P (. . 9.9).

+d

()
L
. 9.9

9. .

353

P S L = 1 ,
R = 0,5 . , , P I0 = 30 /2. .

I. P, .
S
, , (. (9.13)), ,
.
II. , +d (0 R) P.
, ,
(. . 9.9), :
L2
I ( ) = I 0 2
.
(9.108)
+ L2
dJ
(. . 9.9) (. . 9.1.5), :
L
dJ = I ( ) cos ( )2d = I ( )
2d .
(9.109)
2
+ L2
III. J , R, , (9.109) (9.108):
R

J = I ( )
0

+L
2

2d = I 0
0

L3
2

+ L2

( )

d 2 =
2

354

1
.
= 2 I 0L 1
(9.110)
2

R
+1

L2

(9.110) L
P S, R I0 P, :
J = 20 .
2

9.6
X D S s = 2000 .
,
= 50 .
=340 /.
, , = 20 ?

I. , X
(. . 6.1). , , ,
s

, c
.
S
D
,
X
,
. 9.10
(. . 9.1.6).
II. , ,
.
, , max
, (9.51),
c s
max =
,
(9.111)
c s 0

9. .

355

s 0
, c .

, , , ,
:
s 0 = A .
(9.112)
min ,
,
c s
min =
.
(9.113)
c + s 0
III. , , (9.111) (9.113)
:
c
(9.114)
= max min = 2 s 2 s 0 2 .
c s0
(9.112), :
cA
= 2 s 2
.
(9.115)
c A2 2
(9.115) , :
c c s2 + ( )
= s
.
(9.116)
A
,
:
2

1 .
= s 1 +
(9.117)
A

<< s ,
(9.117) ,
2



s
:

356

c
.
(9.118)
2 A s
, , :
3,4 / .

9.7

c, a ,
X, Y
. y = x
XY,
, .

I. ,
X Y .
II.
( (9.8)):
1 (t , x ) = ae x cos( t kx + 0) ,
(9.119)
2 (t , y ) = ae y cos( t ky + 0) ,
(9.120)

e x e y X Y, k =

c
, 0 ,
.
A
y = x XY:
(t , x, y ) = 1 (t , x ) + 2 (t , y ) =
= ae x cos(t kx + 0 ) + ae y cos(t ky + 0 ) =
= ae x cos(t kx + 0 ) + ae y cos(t kx + 0 ) =

= a e x + e y cos(t kx + 0 ) .

(9.121)

9. .

357

y = x
XY e . e x + e y = 2e ,
= a 2e cos(t kx + 0 ) = Ae cos(t kx + 0 ) ,

(9.122)

A :
A = 2a .
(9.123)
(9.122)
X x =


= Ae cos t
(9.124)
+ 0 .
2c

, A
(9.123)
c = 2c .
(9.125)
y = x (9.48) :
A2 2
I = S ( x, t ) T =
c .
(9.126)
2
III. (9.126) A (9.123) c (9.125),

y = x XY:
I = 2a 2 2c .
(9.127)
9.8
S 0 = 1700 D (.
. 9.11).
, u = 6 /. = 340 /.
, .

358

. 9.11

u
c

I. X , , . 9.11. : .
II. (9.51),
:
c d
d =
s ,
(9.128)
c s
s d , c , s
, d , .
.
, , ,
(9.128) :
c d
c u
= 0
1 = 0
.
(9.129)
c
c
, , , (9.128):
c
cu
= 0
2 =1
.
(9.130)
c + s
c+u
(9.130) ,
(

9. .

359

) .
(. 8.10 8), 0 , , 2 , , :
= 0 2 .
(9.131)
III. (9.130) (9.131), :
2u
cu
= 0 0
.
(9.132)
= 0
c+u
c+u
(9.132)
, ,
:
= 0,6 .
9.9
L = 110 , = 7,8 /3
d = 1 .
= 256
,
"" n = 5 .
.

I. , , ,
(. . 9.1.4.). , X
(. . 9.12).

0
. 9.12

360

II.
(. (9.27)
. 9.1.4.)
T
c=
,
(9.133)
S
S :
d 2
S=
.
(9.134)
4

(. (9.133)).
"" n :
L=n

.
(9.135)
2
:
c
(9.136)
= .

III. (9.133) (9.136), :


d 2 L2 2
T=
.
(9.137)
n2

, (9.137), :
T 77,7 .
9.10
n ,
L = 1,7 , ,
c = 340 /.

I. , ,
, .

9. .

361

, , X
(. . 9.13).

. 9.13

II. (. (9.53)):
(t , x) = C cos(kx + 0 ) cos(2t + 0 ) .
(9.138)

, :
(t ,0) = C cos( 0 ) cos(2t + 0 ) = 0 .
(9.139)
, , . . , (9.34), :
(t , x)

=
= 0.
(9.140)
x x = L
x=L

(9.138) x
(9.140), :
(t , x)
= kC sin (kL + 0 ) cos(2t + 0 ) = 0 .
(9.141)
x x = L
III. (9.139) (9.141)
, :
2
(9.142)
cos
L = 0 .
c

,
, , :
c
n = (2n + 1)
, n = 1, 2, 3, .
(9.143)
4L
,
, (. . 9.1).

362

,
, :

4L
n =
L = (2n + 1) n .
(9.144)
4
2n + 1
(9.144) , , , "" .

, (9.143), , :
n = (50 + 100n) .
9.4.
1
k , :
= 0 cos(t x y z ) , .

: k = e x + e y + e z , c =

2 + 2 + 2 , e x , e y , e z

.
2

= 4,0 /3 E = 100 . ,
= 0,01 .

: = e E , e ; = 50 / .
3
,
N = 0,1 , R = 1 h = 2 . , ,

9. .

363

J , .

: J = N

1 + (2 R h ) = 0,07 .
2

4
P = 10 4 = 2 . 0 .
P = 105 ,
= 1,3 /3 = 1,4 .
P
: 0 =
1,9 10 11 .
2 P
5
.
,
= 1%, = 0,42 1
= 50 .
: = (2 ) ln(1 ) 2 0,3 .
6
L = 50 .

20 50 . E = 1,21011 /2,
= 8,9103 /3.
E
1 1
1

: n =
n + = 7.34 n + ;

2l
2

25,7; 33,0; 40,4 47,7 .


7
, , .
,
.

364

: 2 .
8
L = 3 .
,
0 = 700 . c ? , ?
: c = 2 0 L = 4200 /; n = n 0 , n = 2, 3 .
9
L = 1
. 1 10 .
= 2,7103 /3, G = 26 .
G n
: n =
= 3,1 n ; 3,1; 6,2
L
9,3 .
10
, ,
0.
U , ,

,
c.
0
: U = c
.
+0

365

1. . . . . . .
21 , 2003. 432 .
2. . . , . . , . . . . .:
. , 2004. 480 .
3 . . . . .: , 2005. 560 .
4. . . . . . . 1.
. .: / , 2005. 559 .
5. . . . . .: ,
2008. 768 .
6. . . . .
.: , 2009. 576 .
7. . . , . . , . . , . . .
. .: , 1978. 392 .

1. . . . I.
. . . . . .: ; ,
2006. 240 .
2. . . . . .: , 2006.
416 .
3. . . . 1.
. . .
. . . .: - , 2002. 448 .

1. . . . . .
.: - . -, 1980. 160 .
2. . . , . . , . . .
. .
. . . - . -, 1984 . 148 .
3. . . , . . , . . .

. . . . . - . -, 1986 . 76 .

366


1.

.
.
.
. . . .: , 1984. 944 .
2. . . .:
, 1988 1998. 3538 .
3. : . . . .
. . . .: , 1991. 1232 .






.
: ..

15.12.2009 .
6090.16. 23 . . 500 .

. ..
119991, 1, , , . 1, . 2
. ..

You might also like